SlideShare a Scribd company logo
1 of 140
Download to read offline
Sugiyanta (SMAN 48 Jakarta)
sgifis48@gmail.com
08128533491/0817804183
https://t.me/fisikatutorial
antengwibowo@gmail.com
+6281575008063
1. Pengukuran panjang.
Pembahasan:
Angka nol skala nonius berada
antara 3,3 dan 3,4 dan angka
lima skala nonius berimpit
dengan skala utama sehingga
L = 3,3 + 0,05 = 3,35 cm
1) Jangka Sorong
2) Mikrometer Sekrup
Pembahasan:
Sumbu skala nonius berada
sebelah angka 8,0 dan angka
16 skala nonius berimpit
dengan sumbu skala utama
sehingga L= 8,0 + 0,16 = 8,16
mm
Pengukuran sebuah logam berbentuk balok menggu-
nakan jangka sorong diperlihatkan pada gambar.
 Pengukuran panjang
 Pengukuran lebar
 Pengukuran tebal
Hasil pengukuran dan
jumlah angka penting:
 panjang: ……..
 lebar : ……..
 tinggi : ……..
Luas tiga sisi yang
berbeda dari balok:
 sisi 1 :
 sisi 2 :
 sisi 3 :
Volume balok:
1) Analisa Grafik
Perhatikan gambar vektor berikut.
Besar resultan ketiga
vektor tersebut adalah....
A. 10 N
B. 15 N
C. 20 N
D. 25 N
E. 30 N
Pembahasan:
Ketiga vektor diuraikan menjadi
vektor komponen pada arah
sumbu –x dan sumbu –y, yaitu:
x y
F1 +153 0
F2 -153 +15
F3 0 -30
F 0 -15
   
    N
R
F
F
R Y
X
15
15
0
2
2
2
2








 Sebuah partikel bergerak dari posisi awal (0,0)
menuju titik K (3,3) dalam waktu 2 sekon, kemudian
bergerak ke titik L (15,8) dalam waktu 3 sekon.
Maka besar perpindahan elektron dari titik K ke titik
L adalah…
 Besar perpindahan sebuah benda yang bergerak
dalam bidang XY (seperti gambar) adalah… .
   
   
   
2 2
2 2
2 2
.... .... ....
L K L K
r x y
r x x y y
r N
   
   
  
Ditanyakan jarak yg ditempuh ?
Ditanyakan perpindahan yg terjadi ?
Rumus :
• s = vt
• s = (pada grafik v-t : luas
grafik)
Rumus :
• s = vt (pada grafik v-t s = luas grafik)
• di atas sumbu x S bertanda (+)
• di bawah sumbu x S bertanda (-)
 Grafik di bawah mewakili mobil A dan B
Jika keduanya berangkat dari tempat yang sama maka:
1. Percepatan mobil B = 4 m.s-2
2. Kedua mobil bergerak dengan percepatan yang sama
3. Kedua mobil akan bertemu setelah bergerak selama 20 s
4. Kedua mobil akan bertemu setelah menempuh jarak 3200 m
 
1
2
1
2
2
1
2
80 0
.....ms
20 0
..... 0 ....
.......
......meter
B
A B
A Bo B
v
a
t
s s
v t v t a t
t t
t sekon
s

 
  
 

 
 


Grafik berikut mewakili gerak benda P dan benda Q
Jika kedua benda bergerak dari tempat dan waktu yang sama, maka:
1) Kedua benda akan bertemu setelah bergerak selama 20 s
2) Percepatan benda P = 1 m.s-2
3) Percepatan benda Q = 0,5 m.s-2
4) Benda P akan bertemu benda Q setelah menempuh jarak 50 m
Pernyataan yang benar adalah....
Pada lintasan yang lurus dan panjang, 2 buah sepeda, A dan B di kayuh ke arah
yang sama. Sepeda A mulai dari titik S bergerak dari keadaan diam dengan
percepatan tetap 2 m.s-2. Sepeda B melintasi titik S dalam waktu yang bersamaan
dengan sepeda A, namun dengan kecepatan tetap 8 m.s-1. Setelah bergerak
selama 10 s sejak melintasi titik S, jarak antara sepeda A dan B adalah ….
A. 20 m
B. 50 m
C. 60 m
D. 80 m
E. 100 m
Sebuah mobil mengalami perlambatan dari laju 72 km/jam sampai berhenti
dalam waktu 5 detik saat ada rintangan di depannya yang berjarak 55 meter dari
mobil tersebut. Maka panjang lintasan yang ditempuh mobil dalam selang waktu
5 detik adalah.....
Mobil A bergerak dengan kelajuan
tetap 30 km/jam. Satu jam
kemudian Mobil B mulai berangkat
dari tempat yang sama dengan
kelajuan tetap 60 km/jam
mengikuti mobil A. Maka jarak
yang ditempuh saat keduanya
bertemu adalah… .
KuPreTT Solusion bila VB > VA
2 1
2
.....
v v
t
v t

 

2 1
1
2
.....
v v
s v
v t
 

 
 

 
Perhatikan gambar
Waktu yang diperlukan dan jarak yang
ditempuh saat mobil kedua menyusul
mobil pertama… .
A. t = 5 sekon, S2 = 75 meter
B. t = 12,5 sekon, S2 = 187,5 meter
C. t = 12,5 sekon, S2 = 312,5 meter
D. t = 15 sekon, S2 = 225 meter
E. t = 15 sekon, S2 = 450 meter
KuPreTT Solusion:
  
2 1
2
1
0
2
2 ..... ..... 0
.....& .....
s s
at s vt
t t
t s

 
  
 
Prediksi Soal
Tiga buah roda A, B,dan C masing – masing berjari jari 12
cm, 6 cm dan 2 cm seperti pada gambar.
Jika kecepatan sudut roda B
sebesar 12 rad.s-1, maka
kecepatan sudut roda C adalah....
A. 3 rad.s-1
B. 4 rad.s-1
C. 6 rad.s-1
D. 12 rad.s-1
E. 36 rad.s-1
Pembahasan:
4. Gerak Rotasi
1) Hubungan tiga roda
....





a
b
R
R
b
a
c
a
b
b
a
b
a
R
R
v
v





1. Perhatikan gambar rangkaian roda
gigi
Roda gigi penggerak yang memiliki 7 gigi dan
roda gigi yang digerakkan yang memiliki 30
gigi. Bila roda gigi penggerak berputar pada
kecepatan 130 rpm, kecepatan roda gigi yang
digerakkan adalah… .
N1 × ω1 = N2 × ω2
2. Dua buah roda yang seporos. Roda yang
kecil memiliki 16 buah gigi dan roda yang
besar b memiliki 22 gigi. Jika kecepatan
linier roda yang kecil adalah 48 ms-1.
Maka berapa kecepatan linier dari roda
yang besar adalah?
Perhatikan gambar benda yang
bergerak melingkar berikut.
Penyataan yang benar adalah… .
1) Gaya sentripental selalu
berubah
2) Kecepatan liniernya berubah
3) Percepatan sentripetalnya selalu
berubah
4) Jari jari lintasannya selalu
berubah
5) Tegangan talinya selalu
berubah setiap saat
Sebuah kelereng bermassa 100 gr
bergerak pada bidang datar licin
sebuah meja yang tingginya 1.5 m.
saat meninggalkan tepi meja
kecepatan kelereng 4 ms-1 seperti
pada gambar dibawah ini. Kecepatan
kelereng pada saat ketinggian 0.5 m
adalah ….
A. 3 m/s
B. 4 m/s
C. 6 m/s
D. 8 m/s
E. 9 m/s
2 2
2
.....
.....
....
y
x y
h
t
g
v gt
v v v

 
 
  
• Sebuah peluru ditembakkan dari moncong sebuah meriam dengan kelajuan 50
m/s arah mendatar dari atas sebuah bukit, ilustrasi seperti gambar berikut.
Jika percepatan gravitasi bumi adalah 10
m/s2 dan ketinggian bukit 100 m, tentukan :
a. Waktu yang diperlukan peluru untuk
mencapai tanah
b. Jarak mendatar yang dicapai peluru (S)
• Sebuah pesawat terbang bergerak mendatar dengan kecepatan 200 ms-1
melepaskan bom dengan target berjarak AB (2000 m) seperti gambar.
Jika percepatan gravitasi (g)= 10 ms–2,
ketinggian pesawat saat melepaskan bom
adalah … . 2
2
2
....
2
h
g
h
x x g
x
t
x v t v
gx
h
v

 
 
• Perhatikan gambar di bawah ini, A dan B adalah
balok, F adalah gaya, dan T adalah tegangan tali.
T F
A B
Dengan data mA = 2 kg, mB = 3 kg, F = 40 N, sedangkan
bidang permukaannya licin, maka besarnya T adalah....
• Dua benda di tumpuk dan di dorong seperti pada
gambar.
Jika kedua benda m1 dan m2 saling menempel (tidak
ada gesekan), dan koefisien gesekan statis antara
lantai dengan benda m2 =0,4 maka gaya P minimum
agar balok tepat akan bergerak adalah …(g =10 m.s-2)
a) bila koefisien gesekan antara roda-roda troli dengan lantai 0,2
(g=10 ms-2), maka gaya minimum yang diperlukan agar troli dapat
mengangkut kelima barang secara bersama adalah… .
b) bila troli didorong dengan gaya 52 N, balok mana yang bisa
diangkut secara bersamaan?
 
min
.....
....
g total
f m g
F

 

....
....
total
balok total troli
F
m
g
m m m

 
  
Sebuah balok bermassa 5 kg ditarik gaya konstan F = 26 N dari
keadaan diam seperti tampak pada gambar di bawah!
Koefisien gesekan statis dan kinetis antara balok dan lantai masing-
masing 0,4 dan 0,2. Beberapa waktu kemudian balok B ditumpangkan
di atas balok A (anggap tidak ada gesekan antara balok A dan B)
sehingga kedua balok bergerak dengan kecepatan tetap. Besar massa
B adalah ….
A. 5,0 kg
B. 6,5 kg
C. 8,0 kg
D. 13,0 kg
E. 26,0 kg
m1
m2
 Dua buah balok dengan massa ml = 3 kgdan m2 = 1 kg
dihubungkan melalui katrol licin tanpa gesekan seperti pada
gambar di bawah.
• Tiga buah balok M1 = 2 kg, M2 = 3 kg, M3 = 5 kg, dirangakai
seperti gambar, jika koefisien gesek kinetik antara papan
dengan balok M2 dan M3 adalah 0,1 dan 0,2 maka besar
gaya tegang tali T1 dan T2 berturut-turut sebesar….
Jika percepatan gravitasi
10 ms-2, tentukan:
• percepatan benda m1
adalah ….
.....
.



B
A
B
A
m
m
g
m
m
T
2
1 2
( )
....
m g
a
m m
 

• tegangan tali:
Perhatikan sistem katrol di bawah ini !
MC
MB
MA
Massa masing-masing benda mA= 1
kg, mB = 2 kg dan mc = 4 kg. Jika
percepatan gravitasi 10 m/s2 dan µA=
0,2 dan µB = 0,3 tentukan tegangan
tali pada sistem tersebut…
KuPreTT Solusion: gambarlah gaya-gaya yang bekerja!
1. Planet A dan B sama-sama mengelilingi sebuah
bintang, perbandingan jarak planet A dan B
terhadap bintang yang dikelilingi masing-masing
adalah rA dan rB = 4 : 1, tentukan perbandingan
kecepatan planet A dan B dalam mengelilingi
bintang.
2. Sebuah planet berada pada jarak r mengelilingi
matahari dengan kecepatan v. Jika planet x
berada pada jarak 9r dari matahari, berapakan
kecepatan planet tersebut dalam mengelilingi
matahari ?
3. Satelit palapa mengorbit Bumi dengan periode
24 jam. Jika satelit palapa mengorbit planet X
yang memiliki massa sama dengan massa bumi
dan jari-jari orbit 4 kali jari-jari orbit saat
mengorbit bumi, maka satelit mengorbit
planet X dengan periode sebesar….
Tabel berikut adalah data tiga satelit yang mengorbit pada planet
Bumi bermassa (M), rerata jari-jari planet bumi (r), Jaraknya terhadap
satelit terhadap planet Bumi (h),
Dari data tersebut urutan kecepatan orbit satelit dari yang kecil ke
besar adalah… .
7. Momen Gaya
Prediksi Soal
1) Massa batang tidak dapat diabaikan
 Jika massa batang 4 kg, percepatan gravitasi 10 m.s-2 dan titik
A sebagai poros, maka momen gaya total pada sistem batang
tersebut adalah....
2) Massa batang diabaikan
 momen gaya terhadap titik D adalah …..
2
2
1
1 sin r
F
r
F 

 

3
3
2
2
1
1 sin
sin r
F
r
F
r
F 


 


Besar resultan momen gaya di titik C
adalah . . .
Tentukan momen gaya total terhadap
titik perpotongan diagonal bidang
gambar berikut !
Empat gaya bekerja pada sudut-
sudut persegi panjang ABCD seperti
gambar. diketahui AB= 10 cm dan AC
= 4 cm, tanα = ¾, FA=FD=20 N dan
FB = FC = 40 N.
Moment gaya total pada titik perpo-
tongan diagonal bangun tersebut
adalah….
A. 2,16 Nm
B. 2,20 Nm
C. 2,80 Nm
D. 3,36 Nm
E. 4,00 Nm
8. Momen Inertia
Prediksi Soal
1) Momen inertia partikel
Jika sistem diputar pada salah satu sumbunya , maka
momen inersia sistem adalah....
m
3 m
m
a
a 3 a
X
Y
2
2
2
2
1
1
2
1
r
m
r
m
I
I
I
I






Prediksi Soal
2) Momen inertia benda pejal
diketahui massa batang M dan panjang batang L
 diputar di salah satu ujungnya
 diputar di sembarang titik tegak lurus batang
A B
A B
r2
r1
2
3
1
ML
I 
 
3
2
3
1
3 r
r
I L
M


 Letak koordinat titik berat bidang
terhadap titik A (0,0) adalah ….
6 cm
6 cm
√13 cm
9 cm
A B
C
D
E
Ringkasan Materi:
 Letak titik berat persegi panjang pada
titik potong kedua diagonal sudut
 Letak titik berat bangun segitiga sama
sisi pada jarak sepertiga tinggi bangun
 Untuk gabungan beberapa benda
homogen harus dipisahkan terlebih
dahulu, kemudian ditentukan letak titik
berat dan luas masing-masing
selanjutnya dihitung letak titik berat
gabungan.
 Bila benda bukan bidang tetapi berupa
garis atau memiliki volume, maka
luas(A) diganti dengan panjang benda(L)
atau volume benda (V)
Benda X Y Luas A
1 X1= .... Y1= .... A1= ....
2 X2= .... Y2= .... A2= ....
dst .... .... ....
2
1
2
2
1
1
0
....
A
A
X
A
X
A
X




2
1
2
2
1
1
0
....
A
A
Y
A
Y
A
Y




Prediksi soal
• Perhatikan gambar-gambar berikut lalu tentukan letak koordinat titik beratnya
6 6
6
15
7,5
0 15
7,5
x (cm)
y (cm)
Perhatikan gambar berikut.
Agar dikehendaki kedua benda setimbang,
maka didekat benda yang 2 kg perlu
ditambahkan benda lain yang massa dan
posisi benda masing-masing …
A. 19 kg dan 1 m sebelah kiri penumpu
B. 12 kg dan 1,1 m sebelah kiri penumpu
C. 10 kg dan 0,5 m kanan penumpu
D. 10 kg dan 1,0 m kanan penumpu
E. 5,0 kg dan 1,3 m kiri penumpu
Kesetimbangan Benda Tegar
 Syarat setimbang:
sistem tidak bergeser
dan tidak berputar
 
nol
F 

 
nol


Gambar di bawah menunjukkan
4 benda yang berada dalam
keadaan seimbang.
Keseimbangan yang stabil
ditunjukkan oleh gambar …
Tangga dengan massa 40 kg
disandarkan pada dinding seperti
gambar.
 Agar batang berada dalam posisi
stabil, nilai koefisien gesek
minimum adalah ....
 Tinggi maksimum seorang anak
bermassa 40 kg menaiki tangga
sebelum tergelincir adalah… .
3 m
5 m
α
Sebuah batang homogen AC dengan
panjang 4 m dan massanya 50 kg. Pada
ujung C digantungkan beban yang
massanya 20 kg. Batang ditahan oleh tali
T sehingga sistem seimbang. Jika jarak
BC 1 m, maka hitunglah tegangan tali T!
Batang homogen AB dengan berat 10 N. Pada ujung batang
digantung beban seperti pada gambar di bawah.
Jika sistem di atas dalam keadaan seimbang, maka besar tegangan
tali adalah ….
A. 10,0 N
B. 16,7 N
C. 25,0 N
D. 31,3 N
E. 41,7 N
1) Hukum Archimedes
Ringkasan Materi
suatu benda yang dicelupkan sebagian atau seluruhnya ke dalam
zat cair mengalami gaya ke atas yang besarnya sama dengan
berat zat cair yang dipindahkan oleh benda tersebut :
 A (terapung) & B (melayang), besar gaya
angkat = berat benda di udara :
 C (tenggelam), besar gaya angkat = berat benda di udara –
berat benda di dalam air :
 Perbandingan gaya angkat dengan gaya berat di udara :
A
B
C
benda
benda
air
air
benda
air
air
A
V
V
g
m
gV
W
F






air
A W
W
F 

benda
air
A
W
F



Prediksi Soal :
1)Hukum Archimedes massa jenis fluida
Sebuah balok dimasukkan ke dalam wadah A dan wadah B
yang berisi zat cair yang berbeda dan hasilnya seperti gambar
diatas. Maka perbandingan massa jenis zat cair dalam wadah
A, wadah B dan wadah C adalah…
A
B
B
benda
A
benda
B
A
h
h
h
h
h
h


/
/


B
C
C
benda
B
benda
C
B
h
h
h
h
h
h


/
/


A
C
C
benda
A
benda
C
A
h
h
h
h
h
h


/
/


; &
A B C
h h h adalahbagianbenda yangtercelupcairan
Contoh Soal:
1. Sebuah kubus mempunyai rusuk
10 cm, dimasukkan ke dalam
minyak bermassa jenis 800 kg.m-3,
ternyata bagian kubus yang
muncul di permukaan air adalah
setinggi 4 cm seperti gambar
dibawah.
Jika g = 10 m.s-2, maka besarnya
gaya keatas yang bekerja pada
kubus adalah …
2. Sebuah balok massa jenisnya 2.500
kg.m-3 dan ketika di udara beratnya 25
Newton. Tentukan berat balok di dalam
air jika massa jenis air 1000 kg.m-3 dan
percepatan gravitasi bumi 10 m.s-2 !
3. Peti berukuran 30 cm x 40 cm x 50 cm
dengan berat 1.000 N hendak diangkat
dari dasar laut (massa jenis air laut
=1,1 x 103 kg.m3 dan g = 10 ms-2) .
Gaya minimal yang dibutuhkan untuk
mengangkat peti tersebut adalah …
Peti berupa kotak berukuran 100 cm x 150 cm x 200 cm yang
berada di dasar laut hendak diangkat ke permukaaan laut
dengaan cara menariknya dengan rantai. Bila massa jenis rata-
rata peti = 2000 kg.m-3 dan massa jenis air laut = 1000 kg.m-3
,
maka besar gaya angkat minimal yang dibutuhkan untuk
mengangkat peti tersebut adalah …. (abaikan massa rantai).
A. 2,0 kN
B. 3,0 kN
C. 6,0 kN
D. 9,0 kN
E. 18,0 kN
Pernyataan terkait dengan gambar
1) gaya F pada pipa kecil, dongkrak dapat
mengangkat benda yang beratnya 4F pada
pipa besar
2) gaya F pada pipa kecil, dongkrak dapat
mengangkat benda yang beratnya 4W pada
pipa besar
3) tekanan yang ditimbulkan gaya F pada pipa
kecil sama dengan tekanan yang ditimbulkan
gaya berat pada pipa besar
4) tekanan yang ditimbulkan gaya F pada lebih
besar dari tekanan yang ditimbulkan gaya
berat pada pipa besar
Penyataan yang benar adalah …
Dongkrak hidrolik pada
gambar memiliki
perbandingan jari-jari
penampang pipa kecil
dan penampang pipa
besar adalah 1 : 2
Dongkrak hidrolik digunakan untuk mengangkat beban
seperti gambar di samping. Jika jari – jari pada pipa kecil
adalah 2 cm dan jari – jari pipa besar adalah 18 cm, maka
besar gaya minimal yang diperlukan untuk mengangkat
beban 81 kg adalah….(g=10 m.s-2)
Perhatikan tangki bocor
berikut !
Jika air dalam tangki berisi
penuh, maka kecepatan
pancaran air dan jarak
pancaran terjauh adalah....
Ringkasan Materi Azas Torricelli
 Laju pancaran air:
 Waktu yang diperlukan sampai tanah:
 Jarak pancaran:
h1 : tinggi permukaan air terhadap lubang bocoran
h1 : tinggi lubang bocoran terhadap tanah
x
h
h2
v
h1
1
2gh
v 
g
h
t 2
2

2
1
2 xh
h
x 
3) Hukum Bernouli
Ditanyakan R?
Penerapan hukum Bernoulli pada gaya
angkat pesawat terbang.
Bentuk penampang sayap bagian belakang lebih tajam
dan bagian atas melengkung menyebabkan aliran udara
di bagian atas lebih besar dari bagian bawah (v2 > v1).
Dari persamaan Bernoulli :
P1 + ½ .v1
2 + gh1 = P2 + ½ .v2
2 + gh2
Karena Ketinggian kedua sayap dapat dianggap sama
(h1=h2), maka : P1 + ½ .v1
2 = P2 + ½ .v2
2
P1 – P2 = ½  (v2
2 – v1
2) karena F=P x A
Maka : F1 – F2 = ½  A(v2
2 – v1
2) dengan  = massa
jenis udara (kg/m3)
Supaya pesawat dapat terangkat, gaya angkat harus
lebih besar daripada berat pesawat (F1 - F2 > mg). Saat
pesawat dalam keadaan melayang di udara), kelajuan
pesawat harus diatur sedemikian rupa sehingga gaya
angkat sama dengan gaya berat pesawat (F1 - F2 = mg).
1. Gaya angkat pada pesawat terbang dapat
terjadi karena...
1) Tekanan udara di atas sayap lebih besar
dari pada di bawah sayap
2) Kecepatan udara di atas sayap lebih besar
dari pada di bawah sayap
3) Tekanan udara di atas sayap lebih kecil
dari pada di bawah sayap
4) Kecepatan udara di atas sayap lebih kecil
dari pada di bawah sayap
2. Sebuah pesawat dilengkapi dengan dua
buah sayap masing-masing seluas 40 m2
(anggap kerapatan udara adalah 1,2 kg.m-
3). Jika kelajuan aliran udara di atas sayap
adalah 250 m.s-1 dan kelajuan udara di
bawah sayap adalah 200 m.s-1, maka besar
gaya angkat pada pesawat tersebut adalah
… .
1. Sebuah benda bermassa 2 kg mula-mula diam karena pengaruh gaya luar setelah
menempuh jarak 5 m kecepatannya menjadi 6 ms-1. Usaha yang dilakukan oleh gaya luar
adalah ….
W=ΔEk =½m(vt
2-v0
2)
2. Benda bermassa m dari keadaan diam mendapat gaya F sehingga mengalami percepatan a.
Ditanyakan usaha yang dilakukan setelah t sekon atau setelah perpindah sejauh s
3. Sebuah peluru dengan massa 200 gram ditembak vertikal keatas dari permukaan tanah
dengan kecepatan 60 m.s-1. Jika g = 10 m.s-2, maka energi kinetik peluru pada ketinggian
40 m adalah….
4. Bola yang massanya 1 kg dilepas dari titik A menempuh lintasan seperti gambar. Jika
lintasan AB adalah seperempat lingkaran licin dengan jari-jari 75 cm, maka perbandingan
kecepatan bola di titik B dan C adalah ....
Sebuah balok bermassa 2 kg ditarik oleh gaya F
yang membentuk sudut tertentu terhadap
bidang datar sehingga berpindah seperti
gambar di bawah.
Urutan usaha yang dari yang paling kecil ke
besar yang dilakukan oleh gaya F untuk
menarik balok adalah …
No m (kg) F (N)  (o) t (s)
1 1 5 60 10
2 2 5 37 10
3 1 6 53 5
4 2 4 37 10
5 2 5 53 12
1). Sebuah benda yang massanya 4 kg dijatuhkan dari
ketinggian 200 m diatas permukaan tanah tanpa
kecepatan awal. Jika percepatan gravitasi di tempat
tersebut 10 m.s-2, maka usaha yang dilakukan oleh
benda saat mencapai ketinggian 60 m diatas
permukaan tanah adalah....
2) Contoh lain
3 m
β
F
1 m
300
A
B
  ....
1
2 

 h
h
mg
W
  ....
sin 
 s
mg
W 
1) Sebuah benda bermassa sama 4 kg, jatuh bebas dari ketinggian 10 meter
diatas permukaan tanah. Jika percepatan gravitasi 10 m.s-2 , maka
energi kinetik benda saat benda mencapai ketinggian 1 m diatas tanah
sebesar....
2) Sebuah kelereng meluncur tanpa gesekan dari titik A ke titik B seperti
gambar (g = 10 ms-2)
Jika kelereng (m=10 gram)
mula-mula diam, maka energi
kinetik / laju di B adalah …..
  ....
1
2 

 h
h
mg
Ek
 
....
)
(
2
....






A
B
A
B
h
h
g
v
h
h
mg
Ek
Ditanyakan v ?
Balok bermassa m meluncur dari keadaan diam sepanjang bidang
miring licin seperti gambar di bawah ini.
Perbandingan kecepatan balok saat melintasi titik Q dan S adalah ….
A. √1 : √2
B. √1 : √ 4
C. √2 : √5
D. √4 : √1
E. √5 : √2
Perhatikan gambar benda yang
diluncurkan dan menumbuk benda lain
yang sedang diam.
Benda m1 dan m2 mula mula diam. Jika
setelah m1 meluncur terjadi tumbukkan
lenting sempurna antara m1dan m2,
percepatan gravitasi 10 m.s-2 dan
kecepatan benda m1 setelah tumbukkan
adalah 1 m.s-1 maka kecepatan benda
m2 setelah tumbukkan adalah …
Bola P bermassa 2 kg meluncur dari
ketinggian h kemudian menumbuk
benda B yang diam seperti gambar
di bawah.
Jika massa bola Q = 4 kg (g = 10
m.s-2) dan setelah bertumbukkan
bola P berhenti, berapakah besar
energi kinetik bola Q akibat
tumbukkan tersebut?
Q
h = 5 m
P
Silinder pejal dengan jari-jari 5 cm
bermassa 0,25 kg bertranslasi
dengan kelajuan linear 4 m/s.
Tentukan energi kinetik silinder
jika selain bertranslasi silinder
juga berotasi!
Pembahasan
Data dari soal:
m = 0,25 kg
r = 5 cm = 0,05 m
v = 4 m/s
Ek =.....
Jawab:
Sebuah bola pejal terbuat dari besi
menggelinding pada lantai dasar
dengan laju 15 m.s-1. Jika massa
bola 2 kg dan diameternya 40 cm,
maka energi kinetik total bola
adalah… .
A. 90 J
B. 225 J
C. 315 J
D. 400 J
E. 525 J
Bola 1 dan bola 2 di putar masing-masing dengan kecepatan ω1 dan
ω2 seperti pada gambar di bawah.
Bola 1 Bola 2
Perbandingan jari-jari bola 1 dan bola 2 = 2 : 3, sedangkan
perbandingan massa bola 1 dan bola 2 = 1 : 3. Bila energi kinetik
rotasi kedua bola sama, maka perbandingkan kecepatan sudut bola 1
dan 2 adalah ….
Sebuah piringan bermassa 2 kg berbentuk silinder pejal (I = ½ mR2)
dengan jari-jari 0,2 m mula-mula berputar di atas meja dengan
kelajuan sudut 5 rad.s-1. Bila di atas piringan diletakkan cincin
bermassa 0,1 kg dan jari-jari 0,2 meter (I = mR2) dengan poros putar
cincin tepat di atas pusat piringan, maka piringan dan cincin akan
bersama-sama berputar dengan kecepatan sudut .…
A. 0,22 rad.s-1
B. 2,00 rad.s-1
C. 2,20 rad.s-1
D. 4,50 rad.s-1
E. 44,00 rad.s-1
Sebuah bola bermassa 400 gram
yang sedang bergerak mendatar
menumbuk tembok dan dipantulkan
seperti gambar. Besar Impuls yang
dialami bola adalah … .
Kuprett solusion
I = m (v2 – v1)
I =…… (…… – ……)
I = …. Nm
Contoh penerapan seperti ditunjukkan pada
gambar di atas bertujuan untuk ....
A. memperbesar gaya tumbukkan dengan
cara memperbesar waktu tumbukkan
B. meredam gaya tumbukkan dengan cara
memperbesar waktu tumbukkan
C. memperbesar gaya tumbukkan dengan
cara mempersingkat waktu tumbukkan
D. meredam gaya tumbukkan dengan cara
memperkecil waktu tumbukkan
E. memperbesar impuls tumbukkan dengan
cara memperbesar waktu tumbukkan
2a). Sebuah benda yang jatuh bebas menumbuk
lantai seperti gambar, impuls yang diterima
benda adalah …. N.s
Pembahasan:
Tip: hitung dahulu besar v1 dan v2 dengan
menggunakan rumus:
2b). Bola bermassa 0,2 kg jatuh bebas dari ketinggian 1,8 m di
atas lantai dan memantul kembali dengan kecepatan 2 m.s-1
dengan arah berlawanan. Jika percepatan gravitasi 10 m.s-2 ,
maka impuls yang terjadi adalah....
gh
vt 2

-v1
h
1
=
5
m
h
2
=0,8
m
+v1
’
m = 2 kg
  ....
1
2 



 v
v
m
p
I
Ringkasan Materi Tumbukan
Pada peristiwa tumbukan antara dua benda berlaku hukum kekekalan momentum yaitu jumlah
momentum kedua benda sesaat sebelum dan sesudah tumbukan adalah sama
pawal = pakhir
mava +mbvb = mava
1 +mbvb
1
Untuk menentukan kecepatan sesaat masing-masing benda sesudah tumbukan:
 Langkah pertama anggap kecepatan kedua benda sesaat sesudah tumbukan adalah :
 Langkah kedua hitung kecepatan masing benda:
Dimana e : koefiesien restitusi yang besarnya adalah 0 < e < 1
 e=0 , kedua benda bertumbukan tidak lenting sama sekali sehingga keduanya menyatu, maka:
 e=1, kedua benda bertumbukan lenting sempurna, maka:
0 < e < 1, kedua benda bertumbukan lenting sebagian gunakan rumus pada langkah kedua
A B
vA VB
B
A
B
B
A
A
m
m
v
m
v
m
x



 
 
B
B
A
A
v
x
e
x
v
v
x
e
x
v






1
1
B
B
A
A
v
x
v
v
x
v




2
2
1
1
15. Tumbukan
x
v
v B
A 

1
1
Tumbukan Lenting Dua Dimensi
Massa masing-masing partikel adalah mA dan
mB. Dengan menerapkan hukum kekekalan
momentum pada arah sumbu x dan sumbu y,
diperoleh:
 pada sumbu x.
 pada sumbu Y (Voy = nol)
 perbandingan sinus kedua sudut :
Sebuah bola dilemparkan ke tembok
untuk kemudian memantul seperti
tampak pada gambar di bawah.
Jika tumbukan antara bola dan dinding
bersifat lenting sempurna, maka besar
perubahan momentum bola adalah ….
A. nol
B. 2,0 kg m.s-1
C. 4,0 kg m.s-1
D. 8,0 kg m.s-1
E. 16 kg m.s-1
 Bola A dan B berada pada ketinggian yang sama seperti
gambar, masing-masing massanya 12 kg dan 4 kg dilepas-
kan pada ketinggian bidang lengkung, sehingga bola A dan
bola B bertumbukan di O keduanya menyatu bergerak
bersama-sama maka kecepatan kedua bola adalah ....
Tip: hitung dahulu besar v1 dan
v2 dengan menggunakan rumus:
gr
vt 2

...
....
2
....
2
1
1










x
v
v
m
m
v
m
v
m
x
gh
v
gh
v
B
A
B
A
B
B
A
A
B
B
A
A
Kuprett solusion :
 Bola A dan B masing-masing bermassa 1 kg dan 2 kg secara
bersamaan dilepaskan hingga meluncur di atas bidang yang
licin seperti gambar. Kedua bola bertumbukan lenting
sempurna di bidang datar. Kecepatan bola B sesaat setelah
bertumbukan adalah ....
A B
5 m 3,2 m
...
2
...
2
...
2
...
2
1
1













B
B
A
A
B
A
B
B
A
A
B
B
A
A
v
x
v
v
x
v
m
m
v
m
v
m
x
gh
v
gh
v
Kuprett solusion :
 Sebuah gelas kaca memiliki kapasitas 400 cm3, penuh
berisi air pada suhu 200C. Jika gelas beserta air
tersebut dipanaskan hingga suhunya menjadi 700C
(αkaca = 9 x 10-6 0C-1 dan air = 2,1 x 10-4 0C-1), maka
banyak air yang tumpah adalah :
 Sebuah wadah 95 % dari kapasitasnya diisi cairan yang
bersuhu 00C. Wadah terbuat dari bahan dengan
koefisien muai volume 8,0 x 10-5/ 0C. Wadah yang
telah berisi cairan tersebut dipanaskan hingga tepat
pada suhu 100 0C ternyata cairan mulai tumpah.
Koefisien muai volume cairan tersebut adalah …
.....
)
3
( 0 



 t
V
V 

.....
3
0




 

t
V
V
 Logam yang massanya 200 gram bersuhu 900C
dimasukkan ke dalam150 gram air bersuhu 350C.
Kalor jenis logam dan air masing-masing 700 J/kgK
dan 4200 J/kgK. Setelah terjadi keseimbangan termal
suhu air menjadi....
 Sepotong logam 10 gram bersuhu 100 oC dimasukkan
ke dalam 50 gram zat cair yang bersuhu 25 oC,
akhirnya kedua zat mencapai keseimbangan pada
suhu 50 oC. Jika kalor jenis zat cair 20 kal.g-1.oC-1,
maka kalor jenis zat tersebut adalah ... .
   
 
 
c
air
c
air
air
t
t
m
t
t
c
m
air
c
air
air
c
air
c
t
t
c
m
t
t
c
m
Q
Q







logam
logam
logam
logam
logam
logam
logam
Es bersuhu 0oC,dimasukkan ke dalam air bermassa 340 gram suhu
20oC yang ditempatkan pada bejana khusus. Anggap bejana tidak
menyerap/melepaskan kalor. Jika kalor lebur es Les = 80 kal g-1dan
kalor jenis air cair= 1 kal g-1 oC-1 dan semua es mencair dengan suhu
kesetimbangan termal = 5oC, maka besar massa es adalah .…
A. 60 gram
B. 68 gram
C. 75 gram
D. 80 gram
E. 170 gram
Batu es bermassa 400 gram bersuhu -10 oC dipanaskan dengan proses seperti
ditunjukkan oleh grafik hubungan antara jumlah kalor (Q) dan suhu (t) di
bawah.
Jika kalor jenis es = 0,5 kal.gram-1.0C-1, kalor jenis air = 1 kal.gram-1.0C-1 dan
kalor lebur es 80 kal/gram, berapakah kalor total untuk mengubah batu es
menjadi uap seluruhnya?
Laju Perpindahan Kalor
 Konduksi:
 Konveksi:
 Radiasi:
Pertukaran kalor antara dua benda yang suhunya berbeda memiliki
laju perpindahan kalor yang sama.
Prediksi Soal :
Sebuah balok logam yang panjangnya L dipotong menjadi tiga bagian
yang sama, kemudian dicat dengan warna yang berbeda seperti
gambar. Bila ketiga logam dijemur dalam waktu yang sama, bagaimana
kenaikan suhu yang terjadi?
d
T
A
k
H


T
A
h
H 
 .
.
4
T
e
W 

A B C
Perhatikan tabel kalor jenis dari
beberapa logam berikut.
Jika secara bersamaan keempat
logam tersebut di panaskan dalam
waktu yang sama, maka urutan panas
benda dari kurang panas ke lebih
panas adalah …
Dua logam A dan B memiliki luas
penampang dan panjang sama
disambung seperti pada gambar.
Koefisien konduksi termal logam A = 2
kali koefisien konduksi termal logam B,
maka suhu sambungan kedua logam
tersebut adalah ....
A. 20 oC
B. 30 oC
C. 40 oC
D. 60 oC
E. 80 oC
Dua batang P dan Q dengan ukuran yang sama tetapi jenis logam
berbeda dilekatkan seperti pada gambar di bawah. Jika koefisien
konduksi termal P adalah dua kali koefisien konduksi termal Q, maka
besar suhu di titik T adalah ….
A. 600 oC
B. 200 oC
C. 90 oC
D. 60 oC
E. 45 oC
Sifat-sifat gas ideal
1) Gas terdiri atas partikel-partikel padat kecil (mempunyai masa
yang sama) yg bergerak dengan kecepatan tetap dan dengan arah
sembarang.
2) Masing-masing partikel bergerak dalam garis lurus, gerakan
partikel hanya dipengaruhi oleh tumbukan antara masing-masing
partikel atau antara partikel dan dinding. Gaya tarik-menarik
antarpartikel sangat kecil sekali dan dianggap tidak ada
(diabaikan).
3) Tumbukan antara masing-masing partikel atau antara partikel
dengan dinding adalah tumbukan lenting sempurna.
4) Waktu terjadinya tumbukan antarpartikel atau antara partikel
dengan dinding sangat singkat dan bisa diabaikan.
5) Ukuran volume partikel sangat kecil dibandingkan ukuran volume
ruang tempat partikel tersebut bergerak. Berlaku hukum Newton
tentang gerak.
Persamaan gas ideal (mono atomik)
 Hukum Boyle-Gay lussac
Prediksi Soal:
NEk
PV
kT
Ek
NkT
nRT
PV
Nk
nR
c
c
T
PV
3
2
2
3









Tekanan gas ideal dalam ruang tertutup terhadap dinding tabung dirumuskan :
;[p = tekanan (Pa); N = jumlah molekul (partikel) gas; V = volume gas dan Ek adalah
energi kinetik rata-rata molekul (J)]. Berdasarkan persamaan ini pernyataan yang benar
adalah ....
1. tekanan gas terhadap dinding bergantung pada energi kinetik rata-rata molekul
2. energi gas bergantung pada tekanan yang ditimbulkan molekul terhadap dinding
3. suhu gas dalam tabung akan berubah jika tekanan gas berubah
4. jumlah molekul gas berkurang maka volume energi kinetik molekul akan berkurang
5. jika volume gas bertambah maka tekanan akan berkurang
Ek
P V
N
3
2

Gas dengan volume 7 liter pada suhu 27°C dan tekanan 2 atm
dimampatkan hingga tekanan dan suhunya mencapai 6 atm dan suhu
42°C. Volume gas sekarang adalah ….
A. 0,61 liter
B. 2,45 liter
C. 6,10 liter
D. 14,70 liter
E. 24,50 liter
 Hukum Melde (besaran gelombang)
1) Jika panjang dawai yang bergetar (L) 120 cm dan frekuensi
vibrator 50 Hz, maka cepat rambat gelombang pada dawai
adalah … .
2) Jika cepat rambat gelombang pada dawai 20 m/s dan
frekuensi vibrator 50 Hz, maka letak perut ketiga dan simpul
kelima diukur dari ujung pantul adalah … .








 






 



2
1
4
1
2
...
n
Xs
n
Xp
f
v
n
n
...
.
...
2
3





f
v
L



 Persamaan gelombang
Sebuah gelombang berjalan memenuhi persamaan y = 0,2 sin
0,4 π (60t – x), x dan y dalam cm dan t dalam sekon.
Tentukan:
1) amplitudo gelombang 0,2 cm
2) periode gelombang, 12 s
3) panjang gelombang, 5 cm
4) cepat rambat gelombang 60 cm/s
Pernyataan di atas yang benar adalah ….
 
















x
f
t
A
y
kx
t
A
y
2
2
sin
sin
....
.....
2
....
2
1
....
2
....












k
f
T
v
cm
k
sekon
f
T
Hz
f
cm
A









Sebuah gelombang
merambat sepanjang tali
dinyatakan dengan
persamaan ,
, dalam cm. Beda fase
antara dua titik yang
berjarak 5 cm dan 10 cm
dari asal getaran
adalah… .
Perhatikan gambar gelombang yang merambat pada tali di bawah ini.
Beda fase antara titik b dan d serta titik h dan i adalah ….
A. ¼ dan ½
B. ¼ dan ¾
C. ½ dan ¼
D. ½ dan ½
E. ½ dan ¾
Suatu gelombang stasioner
memenuhi persamaan y = 0,1 cos
(4x) sin (20t) dengan x, y dalam
meter dan t dalam sekon. Jika
panjang tali 2 m maka letak simpul
ke lima dari titik pantul adalah …
Ujung seutas tali diikat mati pada
sebuah tiang, sedangkan ujung tali
yang lain digetarkan bolak balik ke
atas dan ke bawah sehingga
terbentuk gelombang stasioner
dengan persamaan:
y = 0,5 sin (5πx) cos π(10t–4) m.
Jarak x terdekat dari ujung pantul
pada saat amplitudo gelombang
stasionernya = 25 cm adalah …
 Dua pipa organa terbuka A dan pipa
organa tertutup B ditiup bersama-
sama. Pipa A menghasilkan nada
dasar yang sama tinggi dengan nada
atas kedua pipa B. Jika panjang pipa
organa A 10 cm, maka panjang pipa
organa B adalah ... .
 Sebuah pipa organa terbuka mengha-
silkan nada atas kedua dengan
frekuensi x Hz, sedangkan pipa
organa tertutup B menghasilkan nada
atas ketiga dengan frekuensi y Hz.
Bila panjang, suhu dan jenis gas
dalam kedua pipa organa sama,
perbandingan y dengan x adalah …
Sebuah pipa organa terbuka ditiup dengan keras sehingga
menghasilkan nada atas ketiga dengan frekuensi 1700 Hz. jika
cepat rambat bunyi di udara 340 m.s-1, maka panjang pipa
organa tersebut adalah ….
A. 50 cm
B. 40 cm
C. 30 cm
D. 25 cm
E. 15 cm
 Efek Doppler
Rumus Efek Doppler:
 Bila hembusan angin dapat
diabaikan:
 Bila hembusan angin tidak
dapat diabaikan:
fp: frekuensi bunyi diterima
pendengar(Hz)
fs : frekuensisumber bunyi(Hz)
v : cepat rambat bunyi di
udara(m/s)
vp: kecepatan pendengar(m/s),
bertanda(+) bila pendengar(P)
menuju sumber bunyi(S)
vs : kecepatan sumber bunyi(m/s),
bertanda (+) bila sumber
bunyi(S) menjauhi
pendengar(P)
va : kecepatan angin(m/s), bertanda
(+) bila bertiup dari arah sumber
bunyi(S) menuju pendengar(P)
s
s
p
p f
v
v
v
v
f 






 


s
a
s
a
p
p f
v
v
v
v
v
v
f 












P
S1 s2
S
P1 P2
S
P
S
P
P
P
P
P
v
v
v
v
f
f



2
1
S
S
P
P
v
v
v
v
f
f



2
1
  
  
S
P
S
P
P
P
v
v
v
v
v
v
v
v
f
f





2
1
Mobil pemadam kebakaran sedang bergerak dengan laju 20 m.s−1 sambil
membunyikan sirine pada frekuensi 400 Hz (cepat rambat bunyi 300
m.s−1). Jika mobil pemadam kebakaran bergerak menjauhi Amri yang
berdiri di tepi jalan, maka perbedaan frekuensi sirine yang didengar oleh
Amri dengan frekuensi sumbernya adalah ....
A. 384 Hz
B. 375 Hz
C. 75 Hz
D. 25 Hz
E. 16 Hz
Prediksi soal
Sebuah mobil ambulan bergerak
dengan kecepatan 36 km.jam-1
melintas di depan pendengar
yang berdiri di tepi jalan sambil
membunyikan sirine dengan
frekuensi 660 Hz. Jika cepat
rambat bunyi di udara 340 m.s-1
maka perbandingan frekuensi
yang didengar pada saat mobil
ambulan menjauhi dan mende-
kati pendengar adalah....
Pembahasan:
Kereta A dan B bergerak saling
mendekati dengan kecepatan
masing-masing 20 ms-1 dan 15
ms-1. Masinis kereta B membu-
nyikan peluit dengan frekuensi
660 Hz, sedangkan kereta A
membunyikan pluit dengan
frekuensi 735 Hz. Jika cepat
rambat bunyi di udara 345 ms-1,
maka orang dalam kereta A akan
mendengar layangan bunyi dengan
frekuensi …
 Intensitas bunyi
Ringkasan Materi:
 Intensitas Bunyi:
 Perbandingan intensitas bunyi antara 2
titik berjarak r1 dan r2 dari sumber bunyi:
 Amplitudo :
Taraf intensitas:
Satu sumber bunyi :
n sumber bunyi : TIn = TI1 + 10 log n
antara 2 titik berjarak r1 dan r2 dari
sumber bunyi:
A
P
I 
2
1 2
2 1
I r
I r
 
  
 
1
2 1
2
20log
r
TI TI
r
 
Prediksi soal
1) Perhatikan gambar
Jika S adalah sumber bunyi yang merambat ke segala
arah dengan daya 100 watt, maka perbandingan
intensitas bunyi pada titik A, B dan C adalah ….
Kuprett solusion :
S
A
B
C
4 m
9 m 12 m
 
9
:
81
:
16
:
:
36
4
9
12
1
:
4
1
:
9
1
:
:
1
:
1
:
1
:
:
2
2
2
2
2
2









C
B
A
C
B
A
C
B
A
C
B
A
I
I
I
x
x
I
I
I
r
r
r
I
I
I
2) Titik P, Q, dan R mempunyai jarak
masing – masing 2 m, 6 m dan 3 m
dari alarm yang sedang berbunyi.
Perbandingan intensitas bunyi di titik
P, Q dan R adalah....
 
....
:
....
:
....
:
:
.....
....
....
....
1
:
....
1
:
....
1
:
:
1
:
1
:
1
:
:
2
2
2
2
2
2









C
B
A
C
B
A
R
Q
P
R
Q
P
I
I
I
x
x
I
I
I
r
r
r
I
I
I
0
log
10 I
I
TI 
1 2
2 1
A r
A r

Bunyi yang didengar pada jarak 3 m dari sumbernya memiliki
intensitas 10-4 W.m2. Taraf intensitas bunyi tersebut saat
didengar pada jarak 300 m dari sumber bunyi adalah …. (Io =
10-12 W.m2)
A. 20 dB
B. 30 dB
C. 40 dB
D. 60 dB
E. 90 dB
Ringkasan Materi Mikroskop
Mata tak berakomodasi
Ciri-ciri: Lensa Okuler menghasilkan dua berkas sinar
sejajar maka mata tak berakomodasi
Rumus:
Mata berakomodasi maksimum
Ciri-ciri: Lensa Okuler menghasilkan dua berkas sinar
yang melebar maka mata berakomodasi maksimum
Rumus:
Ringkasan Materi Teropong
Teropong Bintang
Ciri-ciri: Lensa Okuler menghasilkan berkas sinar
sejajar karena benda yang diamati jaraknya jauh
sehingga mata tak berakomodasi
Rumus:
obyektif okuler
S
1
oby=foby S oku= foku
oku
oby
oby
F
PP
x
S
S
M
1

1
1


oku
oby
oby
F
PP
x
S
S
M
oku
oby
oku
oby
f
f
d
f
f
M



Ringkasan Materi LUP
LUP (kaca pembesar) pada mikroskop dan
teropong berfungsi sebagai Lensa Okuler
Rumus:
 Kekuatan lensa :
 mata berakomodasi maksimum :
 mata tak berakomodasi :
1


f
PP
M
   
cm
m f
f
P
100
1


  "
"
100
cm
dalam
PP
satuan
bila
P
PP
f
PP
M 


1) Sebuah mikroskop jarak fokus okulernya 2,5 cm dan jarak
fokus objektifnya 0,9 cm, digunakan oleh orang bermata
normal (Sn = 25 cm) tanpa berakomodasi dan ternyata
perbesarannya 90 kali . Berarti jarak objektif terhadap lensa
objektifnya adalah ….
Kuprett solusion :
....
5
,
2
25
9
,
0
9
,
0
90



































ob
ob
ok
ob
ob
ob
s
s
f
PP
f
S
f
M
Prediksi soal
Perhatikan pembentukan bayangan
pada teropong bintang berikut.
Perbesaran bayangan yang
dihasilkan adalah....
Kuprett solusion :
Gambar pembentukan bayangan
pada teropong bintang berikut ini :
Agar bayangan akhir benda dapat
diamati pada layar, maka lensa
okuler digeser sejauh… .
Kuprett solusion :
....




oku
oby
oku
oby
f
f
M
f
d
f
Prediksi soal
Sebuah teropong bintang memiliki
lensa obyektif dengan jarak fokus
100 cm dan lensa okuler dengan
jarak fokus 5 cm. Teropong itu
digunakan untuk mengamati benda
langit dengan mata tak berakomo-
dasi. Berapa cm lensa okuler harus
digeser agar bayangan dapat
ditangkap dengan jelas pada sebuah
layar yang dipasang pada jarak 10
cm di belakang okuler dan kemana
arah pergeserannya ?
Pembahasan:
Sebuah teropong bintang memiliki
lensa objektif dengan jarak fokus 5
cm dan jarak focus lensa okulernya
cm. Teropong itu digunakan untuk
mengamati benda langit dengan
mata tak berakomodasi. Agar
bayangan dapat ditangkap dengan
jelas pada sebuah layar yang
dipasang pada jarak 10 cm di
belakang okuler, maka besar jarak
dan arah lensa okuler harus digeser
adalah …
Teropong itu digunakan untuk mengamati
benda langit dengan mata tak berakomodasi,
jalannya sinar seperti gambar
Berapa cm lensa okuler harus digeser agar
bayangan dapat ditangkap dengan jelas pada
sebuah layar yang dipasang pada jarak 10
cm di belakang okuler dan kemana arah
pergeserannya ?
A. 3 cm menjauhi lensa objektif.
B. 3 cm mendekati lensa objektif.
C. 5 cm menjauhi lensa objektif.
D. 5 cm mendekati lensa objektif.
E. 10 cm menjauhi lensa objektif.
3) Sebuah lensa memiliki kekuatan 20 dioptri digunakan
sebagai lup. Jika mata normal menggunakan lup tersebut
dengan berakomodasi maksimum, maka perbesaran
anguler lup adalah …
Kuprett solusion :
   
  ....
1
100
1
100
1







P
PP
f
PP
M
f
f
P
cm
m
Prediksi soal
Seberkas sinar monokromatis dengan panjang
gelombang 6.000 Å (1 Å = 10-10 m) melewati
celah tunggal menghasilkan pola difraksi orde
terang pertama seperti pada gambar. Lebar
celahnya sebesar ....
Kuprett solusion :
sinar
300
....
sin




n
d
Prediksi soal
1) Pada percobaan Young digunakan 2 celah sempit yang berjarak 2 mm satu
sama lain dan layar yang dipasang 1 m dari celah tersebut. Jika dihasilkan
terang kedua pada jarak 0.5 mm dari terang pusat, maka panjang gelombang
cahaya yang digunakan adalah…
Rumus yang diperlukan:
 Interfernsi maksimum : d sin  = m
 jarak pita gelap dan terang berdekatan :
 panjang gelombang cahaya yang digunakan :
2) Pada percobaan young (celah ganda), jika jarak antara dua celahnya dijadikan
dua kali semula, maka jarak antara 2 garis gelap yang berurutan menjadi
A. 4 kali semula
B. 2 kali semula
C. ½ kali semula
D. ¼ kali semula
E. Tetap
....
sin



mL
dy
m
d 

d
L
y
2


3) Seberkas cahaya monokromatis jatuh tegak lurus mengenai
dua buah celah sempit hinga terjadi interferensi. Jika jarak
antara dua celah sempit adalah 4 x 10-2 cm, jarak antara
layar dan celah 0,4 cm serta garis terang ketiga yang
terbentuk berjarak 5 x 10-2 cm dari terang pusat, maka
panjang gelombang cahaya yang digunakan adalah....
A. 1.667 Å
B. 2.873 Å
C. 3.870 Å
D. 4.652 Å
E. 5.400 Å
Pembahasan:
Pada percobaan young (celah ganda), jika jarak antara dua celahnya
dijadikan dua kali semula, maka jarak antara 2 garis gelap yang
berurutan menjadi
A. 4 kali semula
B. 2 kali semula
C. ½ kali semula
D. ¼ kali semula
E. Tetap
 Kotak nomor 1 sampai dengan 5 pada gambar di bawah mewakili jenis-jenis
gelombang elektromagnetik tertentu.
menyebutkan nama gelombang dan manfaatnya/bahayanya !
Perhatikan gambar di bawah ini!
Gambar diatas menunjukkan spektrum
gelombang elektromagnetik. Pernyataan yang
benar berdasarkan gambar di atas adalah ....
A. P merupakan sinar X yang berfungsi untuk
mengetahui bagian dalam tubuh manusia
B. Q merupakan gelombang mikro yang
berfungsi untuk pengamanan bandara
C. R merupakan inframerah untuk penerangan
D. P merupakan gelombang mikro untuk
memanaskan makanan
E. Q merupakan sinar X untuk fisioterapi
Predik Soal
1) Grafik berikut menggambarkan hasil percobaan dari
sebuah pegas yang digantungi beban. Besarnya x
adalah....
2) Grafik hubungan gaya (F) terhadap pertambahan panjang
(x) dari dua pegas A dan pegas B seperti pada gambar di
atas, maka ........
F (kg.m.s-1)
∆X (cm)
A
B
C
6
15
25
5 6

tan
1






k
garis
kemiringan
k
x
F
k
....
5
,
1
2
6
2
2
2
2
1
1





x
x
x
F
x
F
k
10. Grafik berikut menggambarkan hasil percobaan dari
sebuah pegas yang digantungi beban.
Besarnya x adalah....
A. 1,5 cm
B. 2,0 cm
C. 4,0 cm
D. 6,0 cm
E. 8,0 cm
Pembahasan:
....
5
,
1
2
6
2
2
2
2
1
1





x
x
x
F
x
F
k
3) Perhatikan grafik hubungan F dengan pertambahan panjang
pegas ∆L
Pegas yang memiliki konstanta paling besar ditunjukkan
pada huruf …
1. Pada sebuah pegas yang panjangnya 20 cm digantungkan
beban bermassa 400 gram, panjang pegas menjadi 22,5 cm.
Kemudian beban ditarik ke bawah sejauh 10 cm dan
dilepaskan hingga terjadi gerak harmonik. Tentukan :
 Konstanta pegas
 frekuensi getaran pegas
 periode getaran pegas
2. Sebuah bandul sederhana terdiri dari tali yang mempunyai
panjang 40 cm dan pada ujung bawah tali digantungi beban
bermassa 100 gram. Jika percepatan gravitasi 10 m/s2 maka
periode dan frekuensi ayunan bandul sederhana adalah…
3. Lima pegas identik disusun seperti gambar.
Setiap pegas memiliki konstanta 2.000 N.m–1. Jika
pertambahan panjang secara keseluruhan 2 cm, maka
besarnya energi potensial pegas adalah ....
Tiga pegas identikmasing-masing dengan 30 = N.m-1 disusun
seperti tampak pada gambar di samping. Saat beban m ditarik
ke bawah sejauh 5 cm dari posisi keseimbangan untuk
kemudian dilepaskan, pegas melakukan getaran harmonik
sederhana. Berapakah perbandingan energi kinetik dan energi
potensial pegas saat posisi beban m = 2 cm di bawah titik
keseimbangan?
A. 25 : 9
B. 25 : 16
C. 16 : 9
D. 5 : 4
E. 4 : 3
Prediksi soal
Tiga buah muatan disusun seperti pada gambar di bawah
ini :
Gaya Coulomb yang dialami muatan Q3 sebesar ....
Kuprett solusion
Tips: gambar dulu gaya Coulomb yang dialami Q3 karena
Q1 dan Q3
Rumus yg diperlukan:
....
2
2
2
3
2
1
1
3
3
32
31
3





r
Q
Q
k
r
Q
Q
k
F
F
F
F
 Tidak terletak pada satu garis hubung
Besar gaya coulomb yang dialami muatan A sesuai dengan
aturan penjumlahan vektor adalah :
+A
+C -B

FA
FAB
FAC

Cos
F
F
F
F
F AC
AB
AC
AB 2
2
2



 : sudut yang dibentuk oleh
FAB dengan FAC
Prediksi Soal
1) Tiga buah muatan listrik terletak di titik-titik sudut sebuah
segitiga sama sisi ABC. Panjang sisi AB = AC = 5 cm, dan qA = -
10 µC, qB = qC= 10 µC. Besar gaya listrik yang bekerja pada
muatan di titik A adalah….
2) Perhatikan gambar di bawah ini
Jika k = 9 x10 9 Nm – 2 , maka
resultan gaya Coulomb yang
bekerja pada muatan – 10 µC
adalah . . . .
A
B C
6 µC
– 8 µC
30
cm
30 cm
– 10 µC
Dua partikel masing-masing bermuatan qA =
1μC dan qB = 3 μC diletakkan terpisah sejauh
4 cm (k = 9 × 109 Nm2/C2). Besar kuat medan
listrik di tengah-tengah qA dan qB adalah ….
A. 6,75 × 107 N/C D. 3,60 × 107 N/C
B. 4,50 × 107 N/C E. 2,25 × 107 N/C
C. 4,20 × 107 N/C
Pembahasan
Perhatikan arah medan listrik yang terjadi di
tengah-tengah kedua muatan tersebut.
Karena EA dan EB berlawanan arah maka
resultannya merupakan hasil pengurangan
dari kedua medan listrik tersebut.
Perhatikan gambar di bawah ini!
Kuat medan listrik pada titik C sebesar ... (k = 9
× 109 N.m2.C−2).
A. 5,0 × 109 NC−1 D. 3,1 × 1010 NC−1
B. 2,3 × 1010 NC−1 E. 4,0 × 1010 NC−1
C. 2,7 × 1010 NC−1
Pembahasan
Titik C dipengaruhi medan listrik dari muatan A
dan B. Karena muatan A dan B positif maka
arah medan listrik di C dua-duanya ke kanan.
Predikasi Soal
1) Perhatikan gambar rangkaian kapasitor
berikut.
Jika rangkaian tersebut dihubungkan
dengan beda potensial 22 volt, maka besar
muatan yang tersimpan dalam rangkaian
kapasitor tesebut adalah....
Kapasitor
Kapasitas Kapasitor keping sejajar:
Muatan kapasitor: q = CV
Kuat medan listrik:
Energi tersimpan:
Rangkaian Kapasitor:
 Seri, berlaku:
 Paralel, berlaku:
A
q
d
V
E .
1
0
0 





C
q
qV
CV
W
2
2
2
1 2
2



....
....
1
....
1
2
3
6









xV
C
q
C
C
C
total
total
s
total
s
2) Jika X dan Y pada rangkaian di samping
dihubungkan ke batere 24 vot, maka besar
muatan dan potensial pada kapasitor 12 F
adalah ….
3) Perhatikan gambar rangkaian kapasitor di
samping. Besar energi listrik dalam
rangkaian adalah ….
4) Perhatikan grafik hubungan antara muatan
q dan tegangan V sebuah kapasitor keping
sejajar berikut !
Jika jarak antar keping 8,85 cm dano =
8,85 x 10-12 F.m-1, maka luas penampang
keping sejajar adalah …
Prediksi soal
1) Beberapa batere dan resistor
dirangkai seperti gambar. Bila
tegangan jepit pada hambatan
8 Ω adalah 4 volt, maka nilai R
adalah….
R
4 
8 
12 v 3 v
Ringkasan Materi
Hukum ohm: V = I . R
Hukum I Kirchoff
“Jumlah kuat arus listrik yang
masuk ke suatu titik sama
dengan jumlah kuat arus listrik
yang keluar”
Imasuk = Ikeluar
Hukum II Kirchoff
“Di dalam sebuah rangkaian
tertutup, jumlah aljabar gaya
gerak listrik (E) dengan
penurunan tegangan (IR) sama
dengan nol”
E= IR
 
 
....
4
8
8
4
3
12
3
2
1
1
2
1












R
R
R
R
R
R
v
E
E
iR
E
Perhatikan gambar
Bila hambatan 12  diganti
dengan hambatan lain yang
besarnya 4 , perbandingan arus
listrik yang mengalir pada
rangkaian sebelum dan sesudah
diganti adalah…
A. 2 : 1
B. 2 : 3
C. 3 : 2
D. 11 : 12
E. 12 : 11
Beberapa baterai dan resistor
dirangkai seperti gambar.
Pada rangkaian diatas mengalir
arus sebesar I1. Jika hambatan 4
diganti dengan hambatan 19 
maka arus mengalir sebesar I2,
maka nilai I2 adalah....
A. 0,1 I1
B. 0,2 I1
C. 0,3 I1
D. 0,4 I1
E. 0,5 I1
2) Pada gambar rangkaian di bawah, kuat arus listrik mengalir
melalui hambatan 3  adalah ….
Kuprett solusion
Rs = 1 + 3 = …
Rtot = 4 + Rp = …
 Arus yang melalui hambatan yang dirangkai seri :
dan
3) dari gambar di bawah ditanyakan i, VR, VAB ?
Kuprett solusion
...
1 

s
P
R
ixR
i ...
4
2 
 P
ixR
i
A
R3
E3 , r3
E2 , r2
B
C
D
R2
R1
.....
)
(
....
.....
2
1
1
1
3
3
2
2
1
1
3
2
1




















R
r
R
i
E
V
iR
V
r
R
r
R
r
R
E
E
E
R
E
i
AB
R
Prediksi soal
Perhatikan gambar kawat berarus berikut ini.
Jika arus yang mengalir 2 ampere dan titik P berjarak 0,5  cm
dari kawat, maka induksi magnet pada titik P adalah....( µ0 = 4
π x 10-7 Wb A-1m-1 )
Kuprett solusion
Dua buah kawat lurus panjang diletakkan sejajar pada jarak 2
cm satu sama lain dialiri arus seperti pada gambar. Jika arus
yang mengalir pada kedua kawat sama besar 3A, maka induksi
magnetik pada titik P adalah .... (o = 4.10-7 Wb.A-1.m-1)
Kuprett solusion
Tip: dengan aturan tangan kanan untuk
kawat lurus diperoleh arah B yang
ditimbulkan kawat pertama masuk bidang
gambar dan arah B yang ditimbulkan
kawat kedua juga masuk bidang gambar
7
0 4 .10 .... ....
....
2 2 ....
iN x x
B
a x
  
  
....
....
10
.
2
....
10
.
4
....
10
.
2
....
10
.
4
7
7
7
7
2
1 



 



x
x
x
x
B
B
BP




....
10
.
2
2
2
1
1
7
2
1 











 
a
i
a
i
B
B
BP
....
2 2
2
1
1
0



a
ki
a
N
i
BP

Dua buah kawat berarus didekatkan
seperti pada gambar.
Besar induksi magnet di
titik P adalah ....
A. ( 4 + 2π ) .10-6 T
B. ( 2 + 4π ) .10-6 T
C. ( 1 + 2π ) .10-6 T
D. ( 1 + 4π ) .10-6 T
Kuprett solusion
Tip: dengan aturan putaran baut
diperoleh arah B yang ditimbulkan kawat
melingkar masuk bidang gambar dan
arah B yang ditimbulkan kawat lurus
juga masuk bidang gambar.
Bagaimana dengan induksi
magnetik yang ditimbulkan kawat
pada gambar berikut :
i
i
P
Sebuah kawat dialiri arus seperti gambar.
Besar induksi magnet pada titik
P adalah....
A. 0,4 x 10 -5 T
B. 0.8 x 10 -5 T
C. 1.0 x 10 -5 T
D. 1,6 x 10 -5 T
E. 2,4 x 10 -5 T
Pembahasan:
Perhatikan pernyataan yang berkaitan dengan induksi magnet
1) Arus yang mengalir dalam kawat
2) Permeabilitas ruang hampa
3) Luas penampang kawat
4) Massa jenis kawat
Yang mempengaruhi besar induksi magnet disekitar kawat penghantar adalah... .
Gaya magnet (gaya Lorentz)
 Pada kawat berarus listrik: F = Bil sin 
 Pada muatan bergerak: F=BqV sin 
 Pada kawat sejajar:
Arah kedua arus sama saling tarik-menarik
Arah kedua arus berlawanan saling tarik-menarik

i
B
l
A B
U
S
i
x x x x x x x x x x
x x x x x x x x x x
x x x x x x x x x x
x x x x x x x x x x
v
a
l
i
i
F


2
2
1
0

2A 3A
5 cm
arah
arah
32. Gambar berikut menjelaskan arah gaya lorenzt pada
muatan positif yang bergerak dalam medan magnet
homogen.
gambar yang benar
ditunjukkan pada
gambar nomor....
A. ( 1 )
B. ( 2 )
C. ( 3 )
D. ( 4 )
E. ( 5 )
Pembahasan:
Fluks magnetik melalui sebuah loop dinyatakan
dengan persamaan ɸ =( t2 - 8t) x 10-2 wb, dengan
t dalam sekon. Tentukan besar ggl induksi yang
dihasilkan jika jumlah lilitan komparan 400 lilitan
dan t = 7 sekon!
A. 4 volt
B. 12 volt
C. 16 volt
D. 24 volt
E. 30 volt
Fluks magnetik yang dirangkul oleh kumparan 100
lilitan berubah memenuhi persamaan fluks:
 = (3t2 + 2t –1) weber.
Pada saat t = 2 s, besar GGL induksi magnetik di
ujung-ujung kumparan adalah …
Tranformator (menjelaskan teori)
 Bentuk dan Simbol Transformator (Trafo)
terdiri dari kumparan kawat yang terisolasi yaitu kumparan
primer dan kumparan sekunder (biasanya lebih dari satu
lilitan) yang dililitkan pada sebuah besi yang dinamakan
dengan Inti Besi (Core). Sedangkan Inti besi pada terbuat dari
kumpulan lempengan-lempengan besi tipis yang terisolasi
dan ditempel berlapis-lapis untuk mempermudah jalannya
Fluks Magnet yang ditimbulkan oleh arus listrik kumparan
serta untuk mengurangi suhu panas yang ditimbulkan.
 Prinsip Kerja Transformator (Trafo)
1) Ketika kumparan primer dialiri arus AC (bolak-balik) maka akan
menimbulkan fluktuasi medan magnet atau fluks magnetik disekitarnya.
Kekuatan Medan magnet (densitas Fluks Magnet) tersebut dipengaruhi
oleh besarnya arus listrik yang dialirinya. Semakin besar arus listriknya
semakin besar pula medan magnetnya.
2) Fluktuasi medan magnet yang terjadi di sekitar kumparan pertama
(primer) akan menginduksi GGL (Gaya Gerak Listrik) dalam kumparan
kedua (sekunder) dan akan terjadi pelimpahan daya dari kumparan primer
ke kumparan sekunder. Dengan demikian, terjadilah pengubahan taraf
tegangan listrik baik dari tegangan rendah menjadi tegangan yang lebih
tinggi maupun dari tegangan tinggi menjadi tegangan yang rendah.
34. Dalam suatu percobaan tarfo
diperoleh data sebagai
berikut.
A. 1500 volt
B. 2500 volt
C. 3000 volt
D. 4500 volt
E. 6000 volt
Pembahasan:
IP IS VP VS NP NS
4 3 P 300 1200 R
 Rangkaian R murni
 Rangkaian L murni
 Rangkaian C murni
Prediksi soal
1. dari gambar di bawah ini ditanyakan Z atau i
2. Rangkaian RLC seri seperti pada gambar
Bila saklar S ditutup, beda potensial antara titik A dan B adalah....
400 
1 H
10 µF
12 V / 200 rad.s-1
1) Rangkaian RLC disusun seperti pada gambar
Impedansi pada rangkaian
tersebut adalah....
A. 1200 ohm
B. 1000 ohm
C. 500 ohm
D. 200 ohm
E. 125 ohm
2) Di bawah ini adalah diagram fasor dari rangkaian RLC seri yang
dihubungkan dengan tegangan bolak-balik.
Rangkaian yang bersifat kapasitif ditunjukkan pada gambar…
2) Grafik sinusoida berikut ini menunjukan hubungan V- t dan I – t dari
rangkaian seri RLC. Rangkaian yang bersifat kapasitif adalah . .
A. D.
B. E.
E.
V, i
t
0
V
i
V, i
t
0
V
i
V, i
0
V
i
V, i
0
V i
V, i
t
0
V
i
1) Teori atom
a) Teori atom Dalton
Teori atom Dalton dikemukakan berdasarkan hukum
kekekalan massa dan hukum perbandingan tetap. Teori
atom Dalton dikembangkan mulai dari 1803 – 1808, dan
didasarkan atas tiga asumsi, yaitu :
 Setiap unsur kimia tersusun atas partikel – pertikel kecil
yang tidak dapat dihancurkan dan dipisahkan yang
disebut atom (Selama mengalami perubahan kimia, atom tidak
dapat diciptakan dan dimusnakan).
 Semua atom dari suatu unsur mempunyai massa dan sifat yang sama,
tetapi atom – atom dari unsur yang berbeda dengan atom – atom dari
unsur yang lain, baik massa maupun sifat – sifatnya berlainan.
 Dalam senyawa kimiawi, atom – atom dari unsur yang berlainan
melakukan ikatan dengan perbandingan angka sederhana
Kelebihan : kelebihan dari teori atom Daton adalah sebagai berikut :
 Dia adalah orang pertama yang melibatkan kejadian kimiawi seperti halnya
kejadian fisis dalam merumuskan gagasannya tentang atom.
 Dia medasarkan asumsinya pada data kuantitatif, tidak menggunakan
pengamatan kualitatif atau untung – untungan.
Kekurangan : kekurangan dari teori Dalton adalah tidak dapat
menerangkan hubungan antara larutan senyawa dan daya hantar listrik.
 Thompson memperbaiki kelemahan dari teori
atom Dalton dan mengemukakan teori
atomnya yang dikenal dnegan nama teori
atom Thompson.
 Teori ini menyatakan bahwa “atom merupakan bola pejal yang
bemuatan positif dan didalamnya tersebar elektron yang
bermuatan negatif. Model atom Thompson ini dapat anda
bayangkan sebagai tori kismis dimana didalamnya tersebar
elektron. Dengan ungkapan Thompson tersebut, maka dapat
disimpulkan bahwa teori atom Thompson adalah netral.
Kelebihan : kelebihan model atom Thompson, yaitu berhasil
menemukan particle yang lebih kecil dari atom yaitu elektron.
Elektron ini ditemukan melalui percobaan sinar katoda.
Kekeurangan : kekurangan dari model atom Thompson yaitu
tidak dapat menerangkan susunan muatan positif dan negatif
dalam atom tersebut
 Teori atom Rutherford muncul berdasarkan
eksperimen hamburan sinar alfa dari uranium.
Kesimpulannya adalah :
1) Atom tersusun atas inti atom yang bermuatan
positif dan elektron yang bermuatan negatif yang
mengelilinginya
2) Semua proton terkumpul dalam inti atom, dan
menyebabkan inti atom bermuatan positif.
3) Sebagian volume atom adalah ruang kosong. hampir semua massa
atom terpusat pada inti atom yang sangat kecil. Jari – jari atom
sekitar 10-10m sedangkan jari – jari inti atom sekitar 10-15 m.
4) Jumlah proton dalam inti atom sama dengan jumlah elektron yang
mengelilingi inti atom, sedangkan atom bersifat netral.
Kelebihan : kelebihan teori atom ini mampu menjelaskan susunan
positif dan negatif pada ato serta dapat menjelaskan bahwa atom
tidak berbentuk bola pejal dan sebagian besar volumenya
merupakan ruang kosong.
Kekurangan : kekurangan teori ataom ini yaitu tidak dapat
menjelaskan mengapa elektron tidak dapat jatuh kedalam inti atom
Niels Bohr, melakukan percobaan spektrum hidrogen untuk memperbaiki teori atom
Rutherford. Hasil percobaan Bohr :
1) elektron – elektron mengelilingi inti atom yang
terdiri dari Proton dan Neutron pada lintasan –
lintasan tertentu yang disebut kulit eletron atau
tingkat energy. (Tingkat energy paling rendah
adalah kulit elektron yang terletak paling dalam,
semakin keluar maka semakin besar pua nomor
kulitnya dan semakin tinggi tingkat energinya).
2) Menurut teori atom bohr, kulit pertama dalam atom di nama dengan kulit K (n=1),
L(n=2), M,(n=3) dan seterusnya. Dalam model atom bohr ini dikenal istilah konfigurasi
elektron, yaitu susunan elektron pada masing – masing kulit. Konfigurasi pada teori
atom bohr dapat dicari dengan persamaan 2n2 dimana n adalah nomor kulit atom.
Misal, pada kulit L, karena di kulit L ini n = 2, maka jika dimasukkan ke persamaan
adalah 2.22 = 2.4 = 8 Jadi elektron maksimal yang dapat menempati kulit atom kedua
sebanyak 8 elektron.(lebih jelasnya nanti akan dipelajari pada bab konfigurasi elektron)
Kelebihan model atom Bohr
Atom terdiri dari beberapa kulit/subkulit untuk tempat berpindahnya elektron dan
atom membentuk suatu orbit dimana inti atom merupakan positif dan
disekelilingnya terdapat elektron.
Kelemahan model atom Bohr
a. Tidak dapat menjelaskan efek Zeeman dan efek Strack.
b. Tidak dapat menerangkan kejadian-kejadian dalam ikatan kimia dengan baik,
pengaruh medan magnet terhadap atom-atom, dan spektrum atom yang
berelektron lebih banyak.
Model atom mekanika kuantum didasarkan atas :
 Electron bersifat gelombang dan partikel, oleh Louis de Broglie pada
tahun 1923.
 Persamaan gelombang dalam atom, oleh Erwin Schrodinger pada tahun
1926.
 Asas ketidakpastian oleh Werner Heisenberg pada tahun 1927.
Menurut teori mekanika kuantum, elektron tidak bergerak pada lintasan
tertentu. Berdasarkan model atom tersebut, maka model atom mekanika
kuantum adalah sebagai berikut :
 Atom terdiri atas inti atom yang mengandung proton dan neutron. Dan
electron – electron yang mengelilingi inti atom berada pada orbital –
orbital tertentu yang membentuk kulit atom. Hal ini disebut konsep
orbital.
36. Yang merupakan karakteristik dari model atom Bohr
adalah....
A. elektron berputar mengelilingi inti seperti planet
mengelilingi matahari dalam tatasurya
B. Atom bersifat netral sehingga jumlah muatan negatif
sama dengan muatan positif
C. elektron akan memancarkan / menyerap energi jika
elektron mengalami transisi kulit
D. spektrum yang dihasilkan elektron dalam mengitari
inti bersifat kontinyu
E. lintasan elektron dalam mengitari inti berupa
setengah lingkaran
Pembahasan:
Efek foto listrik
 Peristiwa efek fotolistrik yaitu terlepasnya elektron dari permukaan
logam karena logam tersebut disinari cahaya.
 Menurut Einstein energi yang dibawa foton dalam bentuk paket,
seluruhnya diberikan pada elektron, sehingga foton tersebut lenyap.
Oleh karena elektron terikat pada energi ikat tertentu, maka untuk
melepaskan elektron dari energi ikatnya diperlukan energi minimal
sebesar energi ikat elektron yang disebut fungsi kerja (Wo) atau
energi ambang yang besarnya Wo tergantung pada jenis logam yang
digunakan.
 Pernyataan yang berkaitan dengan peristiwa efek foto
listrik adalah....
A. Ikatan atom pada logam yang disinari mudah putus
sehingga elektron dapat terlepas
B. Lepasnya elektron dari ikatan logam disebabkan oleh
cahaya yang memiliki frekeunsi ambang
C. Intensitas cahaya yang digunakan mampu
memutuskan ikatan logam
D. Frekuensi cahaya yang digunakan untuk menyinari
logam harus kecil
E. Energi yang dihasilkan sama dengan energi ambang
Prediksi soal
1. Sebuah roket yang sedang diam dibumi panjangnya 10 m.
Jika roket bergerak dengan kecepatan 0,6 c, maka
menurut pengamat di bumi panjang roket tersebut selama
bergerak adalah ….
Kuprett solusion
2. A dan B adalah sepasang anak kembar. A berkelana di
antariksa dengan pesawat berkecepatan 0,6 c. ketika A
kembali di bumi, menurut B perjalanan A sudah
berlangsung 20 tahun. Maka menurut A perjalanan yang
telah ditempuh memakan waktu …
3. Cepat rambat cahaya di udara c, agar panjang benda yang
diamati susut menjadi 80% dari ukuran benda yang
teramati oleh pengamat yang diam, maka cepat rambat
pengamat yang bergerak terhadap benda tersebut adalah
... .
A. 0,95 c
B. 0,70 c
C. 0,80 c
D. 0,60 c
E. 0,25 c
.....
4
5
10
0




L
L
Prediksi soal
Diketahui massa proton = 1,0078 sma, massa
neutron = 1,0086 sma, massa = 4,002 sma massa =
7,0160 sma, massa = 7,0169 sma, dan 1 sma = 931
MeV. Hitung Energi ikat inti He, Li dan Be !
Kuprett solusion
E = Δm x 931 =(Zmp + (A-Z)mn - minti)x 931 MeV
Langkah penyelesaian:
Inti atom yang terbentuk memenuhi reaksi
fusi berikut ini :
Diketahui : Massa 1H2= 2,0141 sma
Massa 1H3 = 3,0160 sma
Massa = 4,0026 sma
Massa = 1,0086 sma
1 sma = 931,5Mev
Nilai E (energi yang dihasilkan) pada reaksi
fusi tersebut adalah ....
Kuprett solusion
Langkah penyelesaian:
  MeV
m
m
E reaksi
hasil
pereaksi 931




Suatu proses fisi pada Reaktor
Nuklir mengikuti persamaan :
Jika massa neutron 1,009 sma,
massa inti = 235,040 sma, massa
inti = 140,910 sma, massa inti =
91,930 sma dan 1 sma = 931 MeV.
Maka besar energy Q yang
dibebaskan pada reaksi fisi tersebut
adalah …..
39. Perhatikan reaksi inti berikut
0n1 + 12A25  4X9 + 7Y17 + Q
Data massa adalah sebagai berikut.
0n1 = 1,0008 sma 12A25 = 26,0012 sma
4X9 = 9,0500 sma 7Y17 = 17,1500 sma
Jika 1 sma setara dengan 931 MeV, maka dalam reaksi
....
A. Dihasilkan energi 746,7 MeV
B. Diperlukan energi 746,7 MeV
C. Dihaslkan energi 186,2 MeV
D. Diperlukan energi 186,2 MeV
E. Dihasilkan energi 93,1 MeV
Perhatikan reaksi inti berikut ini.
Nilai X, Y dan Z adalah ... .
40. Radio isotop adalah isotop yang bersifat radioaktif
sehingga jejaknya dapat dikenal. Berikut ini adalah
pemanfaatan radio isotop dalam bidang kedokteran:
1) Co-60 : terapi radiasi gamma untuk pengobatan
kanker
2) C-14 : Untuk menentukan umur fosil
3) Galium-67 : untuk pengobatan kelenjar getah bening
4) Sb-124 : untuk mendeteksi adanya kebocoran pipa-
pipa industri.
Pernyataan yang benar adalah ….
A. 1 dan 2
B. 2 dan 3
C. 1 dan 3
D. 2 dan 4
E. 1 dan 4
Perhatikan diagram di bawah ini
N = massa zat radioaktif mula-mula
(dalam gram)
t = waktu selama peluruhan (dalam
tahun)
jika zat radioaktif disimpan selama 6
tahun, maka zat yang tersisa adalah ....
T
t
t
T
t
t
m
m
N
N
T















2
1
2
1
693
,
0
0
0

t (sekon)
m (gram)
100
50
12,5
0 6 9
Perhatikan grafik peluruhan massa suatu zat radioaktif di
bawah
Berdasarkan grafik konstanta peluruhan (λ) zat radioaktif
adalah …
Perhatikan gambar grafik peluruhan berikut ini.
Urutan aktivitas radiasi dari yang kecil ke besar adalah …
https://t.me/fisikatutorial
antengwibowo@gmail.com
+6281575008063
Lampiran:
Berikut ini adalah efek samping yang diteliti pada manusia yang terpapar langsung gelombang mikro, tanpa
harus mengkonsumsi zat makanan yang diradiasi.
 Kerusakan medan energi kehidupan (Orang yang dekat dengan oven microwave saat dioperasikan dalam
jangka lama akan mengalami gangguan medan energi dalam kehidupan mereka).
 Mengurangi energi dari sel (Paralel tegangan seluler individu menggunakan alat akan mengalami penurunan
terutama serum limfatik dalam darah mereka).
 Metabolisme tidak stabil (Energi eksternal yang diaktifkan berpotensi dalam pemanfaatan makanan menjadi
tidak stabil dan mengalami degenerasi).
 Kerusakan sel (Selama proses katabolik pada internal Membran sel ke serum darah, pada proses pencernaan
berpotensi merosot dan tak stabil).
 Sirkuit Otak (Impuls listrik pada persimpangan dari otak berpotensi besar akan merosot dan rusak).
 Sistem Saraf (Saraf/sirkuit listrik akan merosot dan memecah, sementara simetri medan energi yang hilang
dalam neuropleksus (pusat saraf) pada otonom sistem saraf pusat dan belakang).
 Kehilangan Kekuatan Bioelectric (Kekuatan bio listrik pada reticular ascending (netlike) yang mengaktifkan
sistem (sistem yang mengontrol fungsi kesadaran) akan menjadi tak seimbang dan kehilangan arus yang
tepat).
 Kehilangan Energi Vital (Manusia, hewan dan tumbuhan yang berada dalam radius 500m dari peralatan
gelombang micro yang dioperasikan akan mengalami kehilangan energi penting dari kumulatif jangka
panjang . Apalagi hanya dalam beberapa meter).
 Gugup dan mengalami kerusakan sistem limfatik (Dalam jangka panjang “Deposito” residu magnetik akan
terjadi di seluruh sistem saraf dan limfatik. Ketidakseimbangan Hormon.Produksi hormon dan pemeliharaan
keseimbangan hormonal pada laki-laki dan perempuan menjadi tidak stabil dan terganggu).
 Gangguan gelombang otak (Tingkat gangguan pada gelombang pola sinyal alpha, delta dan theta ternyata
lebih tinggi dari normal).
 Gangguan Psikologis (Karena gelombang otak menjadi kusut, efek psikologis yang negatif juga akan
dihasilkan. Hal ini termasuk hilangnya kemampuan memori dan konsentrasi, ambang emosional yang
tertekan, perlambatan proses intellective dan tidur yang terganggu, dengan persentase yang lebih tinggi
dari individu yang mengalami efek kisaran emisi berkelanjutan microwave, baik dari memasak ataupun dari
transmisi stasiun).

More Related Content

Similar to Kuppret.pdf

Similar to Kuppret.pdf (20)

Kls8agussem1ok 140827214110-phpapp02
Kls8agussem1ok 140827214110-phpapp02Kls8agussem1ok 140827214110-phpapp02
Kls8agussem1ok 140827214110-phpapp02
 
Bank Soal Fisika SMA
Bank Soal Fisika SMABank Soal Fisika SMA
Bank Soal Fisika SMA
 
Soal try out fisika alumni 12 okt 2014
Soal try out fisika alumni 12 okt 2014Soal try out fisika alumni 12 okt 2014
Soal try out fisika alumni 12 okt 2014
 
3563729631300103
35637296313001033563729631300103
3563729631300103
 
Empati fis 2
Empati fis 2Empati fis 2
Empati fis 2
 
kinematika gerak
kinematika gerakkinematika gerak
kinematika gerak
 
Fisika uas smk ganjil x tm sip
Fisika uas smk ganjil x tm sipFisika uas smk ganjil x tm sip
Fisika uas smk ganjil x tm sip
 
latso uas kls xi smt 1
latso uas kls xi smt 1latso uas kls xi smt 1
latso uas kls xi smt 1
 
Kisi kisi
Kisi kisiKisi kisi
Kisi kisi
 
Soal osn fisika 2004 nas
Soal osn fisika 2004 nasSoal osn fisika 2004 nas
Soal osn fisika 2004 nas
 
Fisika bab 1
Fisika bab 1Fisika bab 1
Fisika bab 1
 
Soal fisika xi 2
Soal fisika xi 2Soal fisika xi 2
Soal fisika xi 2
 
Soal Latihan UN Fisika SMA 2015 - 2
Soal Latihan UN Fisika SMA 2015 - 2Soal Latihan UN Fisika SMA 2015 - 2
Soal Latihan UN Fisika SMA 2015 - 2
 
Soal Latihan UN Fisika SMA 2015 - 1
Soal Latihan UN Fisika SMA 2015 - 1Soal Latihan UN Fisika SMA 2015 - 1
Soal Latihan UN Fisika SMA 2015 - 1
 
naskahsoalujiansekolahfisika
naskahsoalujiansekolahfisikanaskahsoalujiansekolahfisika
naskahsoalujiansekolahfisika
 
Prediksi UN Fisika SMA 2018
Prediksi UN Fisika SMA 2018Prediksi UN Fisika SMA 2018
Prediksi UN Fisika SMA 2018
 
To un 2015 fisika a
To un 2015 fisika aTo un 2015 fisika a
To un 2015 fisika a
 
Kumpulan soal soal fisika
Kumpulan soal soal  fisikaKumpulan soal soal  fisika
Kumpulan soal soal fisika
 
Kumpulan soal soal un fisika
Kumpulan soal soal  un fisikaKumpulan soal soal  un fisika
Kumpulan soal soal un fisika
 
Kumpulan soal soal un fisika
Kumpulan soal soal  un fisikaKumpulan soal soal  un fisika
Kumpulan soal soal un fisika
 

Recently uploaded

Dampak Pendudukan Jepang.pptx indonesia1
Dampak Pendudukan Jepang.pptx indonesia1Dampak Pendudukan Jepang.pptx indonesia1
Dampak Pendudukan Jepang.pptx indonesia1udin100
 
Aksi nyata disiplin positif Hj. Hasnani (1).pdf
Aksi nyata disiplin positif Hj. Hasnani (1).pdfAksi nyata disiplin positif Hj. Hasnani (1).pdf
Aksi nyata disiplin positif Hj. Hasnani (1).pdfDimanWr1
 
Tugas 1 ABK di SD prodi pendidikan guru sekolah dasar.docx
Tugas 1 ABK di SD prodi pendidikan guru sekolah dasar.docxTugas 1 ABK di SD prodi pendidikan guru sekolah dasar.docx
Tugas 1 ABK di SD prodi pendidikan guru sekolah dasar.docxmawan5982
 
Refleksi Mandiri Modul 1.3 - KANVAS BAGJA.pptx.pptx
Refleksi Mandiri Modul 1.3 - KANVAS BAGJA.pptx.pptxRefleksi Mandiri Modul 1.3 - KANVAS BAGJA.pptx.pptx
Refleksi Mandiri Modul 1.3 - KANVAS BAGJA.pptx.pptxIrfanAudah1
 
tugas 1 anak berkebutihan khusus pelajaran semester 6 jawaban tuton 1.docx
tugas 1 anak berkebutihan khusus pelajaran semester 6 jawaban tuton 1.docxtugas 1 anak berkebutihan khusus pelajaran semester 6 jawaban tuton 1.docx
tugas 1 anak berkebutihan khusus pelajaran semester 6 jawaban tuton 1.docxmawan5982
 
2 KISI-KISI Ujian Sekolah Dasar mata pelajaranPPKn 2024.pdf
2 KISI-KISI Ujian Sekolah Dasar  mata pelajaranPPKn 2024.pdf2 KISI-KISI Ujian Sekolah Dasar  mata pelajaranPPKn 2024.pdf
2 KISI-KISI Ujian Sekolah Dasar mata pelajaranPPKn 2024.pdfsdn3jatiblora
 
PELAKSANAAN + Link2 Materi Pelatihan "Teknik Perhitungan & Verifikasi TKDN & ...
PELAKSANAAN + Link2 Materi Pelatihan "Teknik Perhitungan & Verifikasi TKDN & ...PELAKSANAAN + Link2 Materi Pelatihan "Teknik Perhitungan & Verifikasi TKDN & ...
PELAKSANAAN + Link2 Materi Pelatihan "Teknik Perhitungan & Verifikasi TKDN & ...Kanaidi ken
 
Materi Pertemuan 6 Materi Pertemuan 6.pptx
Materi Pertemuan 6 Materi Pertemuan 6.pptxMateri Pertemuan 6 Materi Pertemuan 6.pptx
Materi Pertemuan 6 Materi Pertemuan 6.pptxRezaWahyuni6
 
MODUL AJAR MATEMATIKA KELAS 6 KURIKULUM MERDEKA
MODUL AJAR MATEMATIKA KELAS 6 KURIKULUM MERDEKAMODUL AJAR MATEMATIKA KELAS 6 KURIKULUM MERDEKA
MODUL AJAR MATEMATIKA KELAS 6 KURIKULUM MERDEKAAndiCoc
 
PELAKSANAAN + Link2 Materi TRAINING "Effective SUPERVISORY & LEADERSHIP Sk...
PELAKSANAAN  + Link2 Materi TRAINING "Effective  SUPERVISORY &  LEADERSHIP Sk...PELAKSANAAN  + Link2 Materi TRAINING "Effective  SUPERVISORY &  LEADERSHIP Sk...
PELAKSANAAN + Link2 Materi TRAINING "Effective SUPERVISORY & LEADERSHIP Sk...Kanaidi ken
 
AKSI NYATA MODUL 1.2-1 untuk pendidikan guru penggerak.pptx
AKSI NYATA MODUL 1.2-1 untuk pendidikan guru penggerak.pptxAKSI NYATA MODUL 1.2-1 untuk pendidikan guru penggerak.pptx
AKSI NYATA MODUL 1.2-1 untuk pendidikan guru penggerak.pptxWirionSembiring2
 
soal AKM Mata Pelajaran PPKN kelas .pptx
soal AKM Mata Pelajaran PPKN kelas .pptxsoal AKM Mata Pelajaran PPKN kelas .pptx
soal AKM Mata Pelajaran PPKN kelas .pptxazhari524
 
Modul Ajar Pendidikan Pancasila Kelas 5 Fase C
Modul Ajar Pendidikan Pancasila Kelas 5 Fase CModul Ajar Pendidikan Pancasila Kelas 5 Fase C
Modul Ajar Pendidikan Pancasila Kelas 5 Fase CAbdiera
 
TUGAS GURU PENGGERAK Aksi Nyata Modul 1.1.pdf
TUGAS GURU PENGGERAK Aksi Nyata Modul 1.1.pdfTUGAS GURU PENGGERAK Aksi Nyata Modul 1.1.pdf
TUGAS GURU PENGGERAK Aksi Nyata Modul 1.1.pdfElaAditya
 
Laporan Guru Piket untuk Pengisian RHK Guru Pengelolaan KInerja Guru di PMM
Laporan Guru Piket untuk Pengisian RHK Guru Pengelolaan KInerja Guru di PMMLaporan Guru Piket untuk Pengisian RHK Guru Pengelolaan KInerja Guru di PMM
Laporan Guru Piket untuk Pengisian RHK Guru Pengelolaan KInerja Guru di PMMmulyadia43
 
REFLEKSI MANDIRI_Prakarsa Perubahan BAGJA Modul 1.3.pdf
REFLEKSI MANDIRI_Prakarsa Perubahan BAGJA Modul 1.3.pdfREFLEKSI MANDIRI_Prakarsa Perubahan BAGJA Modul 1.3.pdf
REFLEKSI MANDIRI_Prakarsa Perubahan BAGJA Modul 1.3.pdfirwanabidin08
 
PPT Integrasi Islam & Ilmu Pengetahuan.pptx
PPT Integrasi Islam & Ilmu Pengetahuan.pptxPPT Integrasi Islam & Ilmu Pengetahuan.pptx
PPT Integrasi Islam & Ilmu Pengetahuan.pptxnerow98
 
Modul 1.2.a.8 Koneksi antar materi 1.2.pdf
Modul 1.2.a.8 Koneksi antar materi 1.2.pdfModul 1.2.a.8 Koneksi antar materi 1.2.pdf
Modul 1.2.a.8 Koneksi antar materi 1.2.pdfSitiJulaeha820399
 
aku-dan-kebutuhanku-Kelas 4 SD Mapel IPAS
aku-dan-kebutuhanku-Kelas 4 SD Mapel IPASaku-dan-kebutuhanku-Kelas 4 SD Mapel IPAS
aku-dan-kebutuhanku-Kelas 4 SD Mapel IPASreskosatrio1
 
Kelompok 1_Karakteristik negara jepang.pdf
Kelompok 1_Karakteristik negara jepang.pdfKelompok 1_Karakteristik negara jepang.pdf
Kelompok 1_Karakteristik negara jepang.pdfCloverash1
 

Recently uploaded (20)

Dampak Pendudukan Jepang.pptx indonesia1
Dampak Pendudukan Jepang.pptx indonesia1Dampak Pendudukan Jepang.pptx indonesia1
Dampak Pendudukan Jepang.pptx indonesia1
 
Aksi nyata disiplin positif Hj. Hasnani (1).pdf
Aksi nyata disiplin positif Hj. Hasnani (1).pdfAksi nyata disiplin positif Hj. Hasnani (1).pdf
Aksi nyata disiplin positif Hj. Hasnani (1).pdf
 
Tugas 1 ABK di SD prodi pendidikan guru sekolah dasar.docx
Tugas 1 ABK di SD prodi pendidikan guru sekolah dasar.docxTugas 1 ABK di SD prodi pendidikan guru sekolah dasar.docx
Tugas 1 ABK di SD prodi pendidikan guru sekolah dasar.docx
 
Refleksi Mandiri Modul 1.3 - KANVAS BAGJA.pptx.pptx
Refleksi Mandiri Modul 1.3 - KANVAS BAGJA.pptx.pptxRefleksi Mandiri Modul 1.3 - KANVAS BAGJA.pptx.pptx
Refleksi Mandiri Modul 1.3 - KANVAS BAGJA.pptx.pptx
 
tugas 1 anak berkebutihan khusus pelajaran semester 6 jawaban tuton 1.docx
tugas 1 anak berkebutihan khusus pelajaran semester 6 jawaban tuton 1.docxtugas 1 anak berkebutihan khusus pelajaran semester 6 jawaban tuton 1.docx
tugas 1 anak berkebutihan khusus pelajaran semester 6 jawaban tuton 1.docx
 
2 KISI-KISI Ujian Sekolah Dasar mata pelajaranPPKn 2024.pdf
2 KISI-KISI Ujian Sekolah Dasar  mata pelajaranPPKn 2024.pdf2 KISI-KISI Ujian Sekolah Dasar  mata pelajaranPPKn 2024.pdf
2 KISI-KISI Ujian Sekolah Dasar mata pelajaranPPKn 2024.pdf
 
PELAKSANAAN + Link2 Materi Pelatihan "Teknik Perhitungan & Verifikasi TKDN & ...
PELAKSANAAN + Link2 Materi Pelatihan "Teknik Perhitungan & Verifikasi TKDN & ...PELAKSANAAN + Link2 Materi Pelatihan "Teknik Perhitungan & Verifikasi TKDN & ...
PELAKSANAAN + Link2 Materi Pelatihan "Teknik Perhitungan & Verifikasi TKDN & ...
 
Materi Pertemuan 6 Materi Pertemuan 6.pptx
Materi Pertemuan 6 Materi Pertemuan 6.pptxMateri Pertemuan 6 Materi Pertemuan 6.pptx
Materi Pertemuan 6 Materi Pertemuan 6.pptx
 
MODUL AJAR MATEMATIKA KELAS 6 KURIKULUM MERDEKA
MODUL AJAR MATEMATIKA KELAS 6 KURIKULUM MERDEKAMODUL AJAR MATEMATIKA KELAS 6 KURIKULUM MERDEKA
MODUL AJAR MATEMATIKA KELAS 6 KURIKULUM MERDEKA
 
PELAKSANAAN + Link2 Materi TRAINING "Effective SUPERVISORY & LEADERSHIP Sk...
PELAKSANAAN  + Link2 Materi TRAINING "Effective  SUPERVISORY &  LEADERSHIP Sk...PELAKSANAAN  + Link2 Materi TRAINING "Effective  SUPERVISORY &  LEADERSHIP Sk...
PELAKSANAAN + Link2 Materi TRAINING "Effective SUPERVISORY & LEADERSHIP Sk...
 
AKSI NYATA MODUL 1.2-1 untuk pendidikan guru penggerak.pptx
AKSI NYATA MODUL 1.2-1 untuk pendidikan guru penggerak.pptxAKSI NYATA MODUL 1.2-1 untuk pendidikan guru penggerak.pptx
AKSI NYATA MODUL 1.2-1 untuk pendidikan guru penggerak.pptx
 
soal AKM Mata Pelajaran PPKN kelas .pptx
soal AKM Mata Pelajaran PPKN kelas .pptxsoal AKM Mata Pelajaran PPKN kelas .pptx
soal AKM Mata Pelajaran PPKN kelas .pptx
 
Modul Ajar Pendidikan Pancasila Kelas 5 Fase C
Modul Ajar Pendidikan Pancasila Kelas 5 Fase CModul Ajar Pendidikan Pancasila Kelas 5 Fase C
Modul Ajar Pendidikan Pancasila Kelas 5 Fase C
 
TUGAS GURU PENGGERAK Aksi Nyata Modul 1.1.pdf
TUGAS GURU PENGGERAK Aksi Nyata Modul 1.1.pdfTUGAS GURU PENGGERAK Aksi Nyata Modul 1.1.pdf
TUGAS GURU PENGGERAK Aksi Nyata Modul 1.1.pdf
 
Laporan Guru Piket untuk Pengisian RHK Guru Pengelolaan KInerja Guru di PMM
Laporan Guru Piket untuk Pengisian RHK Guru Pengelolaan KInerja Guru di PMMLaporan Guru Piket untuk Pengisian RHK Guru Pengelolaan KInerja Guru di PMM
Laporan Guru Piket untuk Pengisian RHK Guru Pengelolaan KInerja Guru di PMM
 
REFLEKSI MANDIRI_Prakarsa Perubahan BAGJA Modul 1.3.pdf
REFLEKSI MANDIRI_Prakarsa Perubahan BAGJA Modul 1.3.pdfREFLEKSI MANDIRI_Prakarsa Perubahan BAGJA Modul 1.3.pdf
REFLEKSI MANDIRI_Prakarsa Perubahan BAGJA Modul 1.3.pdf
 
PPT Integrasi Islam & Ilmu Pengetahuan.pptx
PPT Integrasi Islam & Ilmu Pengetahuan.pptxPPT Integrasi Islam & Ilmu Pengetahuan.pptx
PPT Integrasi Islam & Ilmu Pengetahuan.pptx
 
Modul 1.2.a.8 Koneksi antar materi 1.2.pdf
Modul 1.2.a.8 Koneksi antar materi 1.2.pdfModul 1.2.a.8 Koneksi antar materi 1.2.pdf
Modul 1.2.a.8 Koneksi antar materi 1.2.pdf
 
aku-dan-kebutuhanku-Kelas 4 SD Mapel IPAS
aku-dan-kebutuhanku-Kelas 4 SD Mapel IPASaku-dan-kebutuhanku-Kelas 4 SD Mapel IPAS
aku-dan-kebutuhanku-Kelas 4 SD Mapel IPAS
 
Kelompok 1_Karakteristik negara jepang.pdf
Kelompok 1_Karakteristik negara jepang.pdfKelompok 1_Karakteristik negara jepang.pdf
Kelompok 1_Karakteristik negara jepang.pdf
 

Kuppret.pdf

  • 1. Sugiyanta (SMAN 48 Jakarta) sgifis48@gmail.com 08128533491/0817804183 https://t.me/fisikatutorial antengwibowo@gmail.com +6281575008063
  • 2. 1. Pengukuran panjang. Pembahasan: Angka nol skala nonius berada antara 3,3 dan 3,4 dan angka lima skala nonius berimpit dengan skala utama sehingga L = 3,3 + 0,05 = 3,35 cm 1) Jangka Sorong 2) Mikrometer Sekrup Pembahasan: Sumbu skala nonius berada sebelah angka 8,0 dan angka 16 skala nonius berimpit dengan sumbu skala utama sehingga L= 8,0 + 0,16 = 8,16 mm
  • 3. Pengukuran sebuah logam berbentuk balok menggu- nakan jangka sorong diperlihatkan pada gambar.  Pengukuran panjang  Pengukuran lebar  Pengukuran tebal Hasil pengukuran dan jumlah angka penting:  panjang: ……..  lebar : ……..  tinggi : …….. Luas tiga sisi yang berbeda dari balok:  sisi 1 :  sisi 2 :  sisi 3 : Volume balok:
  • 4. 1) Analisa Grafik Perhatikan gambar vektor berikut. Besar resultan ketiga vektor tersebut adalah.... A. 10 N B. 15 N C. 20 N D. 25 N E. 30 N Pembahasan: Ketiga vektor diuraikan menjadi vektor komponen pada arah sumbu –x dan sumbu –y, yaitu: x y F1 +153 0 F2 -153 +15 F3 0 -30 F 0 -15         N R F F R Y X 15 15 0 2 2 2 2        
  • 5.  Sebuah partikel bergerak dari posisi awal (0,0) menuju titik K (3,3) dalam waktu 2 sekon, kemudian bergerak ke titik L (15,8) dalam waktu 3 sekon. Maka besar perpindahan elektron dari titik K ke titik L adalah…  Besar perpindahan sebuah benda yang bergerak dalam bidang XY (seperti gambar) adalah… .             2 2 2 2 2 2 .... .... .... L K L K r x y r x x y y r N           
  • 6. Ditanyakan jarak yg ditempuh ? Ditanyakan perpindahan yg terjadi ? Rumus : • s = vt • s = (pada grafik v-t : luas grafik) Rumus : • s = vt (pada grafik v-t s = luas grafik) • di atas sumbu x S bertanda (+) • di bawah sumbu x S bertanda (-)
  • 7.  Grafik di bawah mewakili mobil A dan B Jika keduanya berangkat dari tempat yang sama maka: 1. Percepatan mobil B = 4 m.s-2 2. Kedua mobil bergerak dengan percepatan yang sama 3. Kedua mobil akan bertemu setelah bergerak selama 20 s 4. Kedua mobil akan bertemu setelah menempuh jarak 3200 m   1 2 1 2 2 1 2 80 0 .....ms 20 0 ..... 0 .... ....... ......meter B A B A Bo B v a t s s v t v t a t t t t sekon s               
  • 8. Grafik berikut mewakili gerak benda P dan benda Q Jika kedua benda bergerak dari tempat dan waktu yang sama, maka: 1) Kedua benda akan bertemu setelah bergerak selama 20 s 2) Percepatan benda P = 1 m.s-2 3) Percepatan benda Q = 0,5 m.s-2 4) Benda P akan bertemu benda Q setelah menempuh jarak 50 m Pernyataan yang benar adalah....
  • 9. Pada lintasan yang lurus dan panjang, 2 buah sepeda, A dan B di kayuh ke arah yang sama. Sepeda A mulai dari titik S bergerak dari keadaan diam dengan percepatan tetap 2 m.s-2. Sepeda B melintasi titik S dalam waktu yang bersamaan dengan sepeda A, namun dengan kecepatan tetap 8 m.s-1. Setelah bergerak selama 10 s sejak melintasi titik S, jarak antara sepeda A dan B adalah …. A. 20 m B. 50 m C. 60 m D. 80 m E. 100 m Sebuah mobil mengalami perlambatan dari laju 72 km/jam sampai berhenti dalam waktu 5 detik saat ada rintangan di depannya yang berjarak 55 meter dari mobil tersebut. Maka panjang lintasan yang ditempuh mobil dalam selang waktu 5 detik adalah.....
  • 10. Mobil A bergerak dengan kelajuan tetap 30 km/jam. Satu jam kemudian Mobil B mulai berangkat dari tempat yang sama dengan kelajuan tetap 60 km/jam mengikuti mobil A. Maka jarak yang ditempuh saat keduanya bertemu adalah… . KuPreTT Solusion bila VB > VA 2 1 2 ..... v v t v t     2 1 1 2 ..... v v s v v t           Perhatikan gambar Waktu yang diperlukan dan jarak yang ditempuh saat mobil kedua menyusul mobil pertama… . A. t = 5 sekon, S2 = 75 meter B. t = 12,5 sekon, S2 = 187,5 meter C. t = 12,5 sekon, S2 = 312,5 meter D. t = 15 sekon, S2 = 225 meter E. t = 15 sekon, S2 = 450 meter KuPreTT Solusion:    2 1 2 1 0 2 2 ..... ..... 0 .....& ..... s s at s vt t t t s        
  • 11. Prediksi Soal Tiga buah roda A, B,dan C masing – masing berjari jari 12 cm, 6 cm dan 2 cm seperti pada gambar. Jika kecepatan sudut roda B sebesar 12 rad.s-1, maka kecepatan sudut roda C adalah.... A. 3 rad.s-1 B. 4 rad.s-1 C. 6 rad.s-1 D. 12 rad.s-1 E. 36 rad.s-1 Pembahasan: 4. Gerak Rotasi 1) Hubungan tiga roda ....      a b R R b a c a b b a b a R R v v     
  • 12. 1. Perhatikan gambar rangkaian roda gigi Roda gigi penggerak yang memiliki 7 gigi dan roda gigi yang digerakkan yang memiliki 30 gigi. Bila roda gigi penggerak berputar pada kecepatan 130 rpm, kecepatan roda gigi yang digerakkan adalah… . N1 × ω1 = N2 × ω2 2. Dua buah roda yang seporos. Roda yang kecil memiliki 16 buah gigi dan roda yang besar b memiliki 22 gigi. Jika kecepatan linier roda yang kecil adalah 48 ms-1. Maka berapa kecepatan linier dari roda yang besar adalah? Perhatikan gambar benda yang bergerak melingkar berikut. Penyataan yang benar adalah… . 1) Gaya sentripental selalu berubah 2) Kecepatan liniernya berubah 3) Percepatan sentripetalnya selalu berubah 4) Jari jari lintasannya selalu berubah 5) Tegangan talinya selalu berubah setiap saat
  • 13. Sebuah kelereng bermassa 100 gr bergerak pada bidang datar licin sebuah meja yang tingginya 1.5 m. saat meninggalkan tepi meja kecepatan kelereng 4 ms-1 seperti pada gambar dibawah ini. Kecepatan kelereng pada saat ketinggian 0.5 m adalah …. A. 3 m/s B. 4 m/s C. 6 m/s D. 8 m/s E. 9 m/s 2 2 2 ..... ..... .... y x y h t g v gt v v v        
  • 14. • Sebuah peluru ditembakkan dari moncong sebuah meriam dengan kelajuan 50 m/s arah mendatar dari atas sebuah bukit, ilustrasi seperti gambar berikut. Jika percepatan gravitasi bumi adalah 10 m/s2 dan ketinggian bukit 100 m, tentukan : a. Waktu yang diperlukan peluru untuk mencapai tanah b. Jarak mendatar yang dicapai peluru (S) • Sebuah pesawat terbang bergerak mendatar dengan kecepatan 200 ms-1 melepaskan bom dengan target berjarak AB (2000 m) seperti gambar. Jika percepatan gravitasi (g)= 10 ms–2, ketinggian pesawat saat melepaskan bom adalah … . 2 2 2 .... 2 h g h x x g x t x v t v gx h v     
  • 15. • Perhatikan gambar di bawah ini, A dan B adalah balok, F adalah gaya, dan T adalah tegangan tali. T F A B Dengan data mA = 2 kg, mB = 3 kg, F = 40 N, sedangkan bidang permukaannya licin, maka besarnya T adalah.... • Dua benda di tumpuk dan di dorong seperti pada gambar. Jika kedua benda m1 dan m2 saling menempel (tidak ada gesekan), dan koefisien gesekan statis antara lantai dengan benda m2 =0,4 maka gaya P minimum agar balok tepat akan bergerak adalah …(g =10 m.s-2)
  • 16. a) bila koefisien gesekan antara roda-roda troli dengan lantai 0,2 (g=10 ms-2), maka gaya minimum yang diperlukan agar troli dapat mengangkut kelima barang secara bersama adalah… . b) bila troli didorong dengan gaya 52 N, balok mana yang bisa diangkut secara bersamaan?   min ..... .... g total f m g F     .... .... total balok total troli F m g m m m      
  • 17. Sebuah balok bermassa 5 kg ditarik gaya konstan F = 26 N dari keadaan diam seperti tampak pada gambar di bawah! Koefisien gesekan statis dan kinetis antara balok dan lantai masing- masing 0,4 dan 0,2. Beberapa waktu kemudian balok B ditumpangkan di atas balok A (anggap tidak ada gesekan antara balok A dan B) sehingga kedua balok bergerak dengan kecepatan tetap. Besar massa B adalah …. A. 5,0 kg B. 6,5 kg C. 8,0 kg D. 13,0 kg E. 26,0 kg
  • 18. m1 m2  Dua buah balok dengan massa ml = 3 kgdan m2 = 1 kg dihubungkan melalui katrol licin tanpa gesekan seperti pada gambar di bawah. • Tiga buah balok M1 = 2 kg, M2 = 3 kg, M3 = 5 kg, dirangakai seperti gambar, jika koefisien gesek kinetik antara papan dengan balok M2 dan M3 adalah 0,1 dan 0,2 maka besar gaya tegang tali T1 dan T2 berturut-turut sebesar…. Jika percepatan gravitasi 10 ms-2, tentukan: • percepatan benda m1 adalah …. ..... .    B A B A m m g m m T 2 1 2 ( ) .... m g a m m    • tegangan tali:
  • 19. Perhatikan sistem katrol di bawah ini ! MC MB MA Massa masing-masing benda mA= 1 kg, mB = 2 kg dan mc = 4 kg. Jika percepatan gravitasi 10 m/s2 dan µA= 0,2 dan µB = 0,3 tentukan tegangan tali pada sistem tersebut… KuPreTT Solusion: gambarlah gaya-gaya yang bekerja!
  • 20. 1. Planet A dan B sama-sama mengelilingi sebuah bintang, perbandingan jarak planet A dan B terhadap bintang yang dikelilingi masing-masing adalah rA dan rB = 4 : 1, tentukan perbandingan kecepatan planet A dan B dalam mengelilingi bintang. 2. Sebuah planet berada pada jarak r mengelilingi matahari dengan kecepatan v. Jika planet x berada pada jarak 9r dari matahari, berapakan kecepatan planet tersebut dalam mengelilingi matahari ? 3. Satelit palapa mengorbit Bumi dengan periode 24 jam. Jika satelit palapa mengorbit planet X yang memiliki massa sama dengan massa bumi dan jari-jari orbit 4 kali jari-jari orbit saat mengorbit bumi, maka satelit mengorbit planet X dengan periode sebesar….
  • 21. Tabel berikut adalah data tiga satelit yang mengorbit pada planet Bumi bermassa (M), rerata jari-jari planet bumi (r), Jaraknya terhadap satelit terhadap planet Bumi (h), Dari data tersebut urutan kecepatan orbit satelit dari yang kecil ke besar adalah… .
  • 22. 7. Momen Gaya Prediksi Soal 1) Massa batang tidak dapat diabaikan  Jika massa batang 4 kg, percepatan gravitasi 10 m.s-2 dan titik A sebagai poros, maka momen gaya total pada sistem batang tersebut adalah.... 2) Massa batang diabaikan  momen gaya terhadap titik D adalah ….. 2 2 1 1 sin r F r F      3 3 2 2 1 1 sin sin r F r F r F       
  • 23. Besar resultan momen gaya di titik C adalah . . .
  • 24. Tentukan momen gaya total terhadap titik perpotongan diagonal bidang gambar berikut ! Empat gaya bekerja pada sudut- sudut persegi panjang ABCD seperti gambar. diketahui AB= 10 cm dan AC = 4 cm, tanα = ¾, FA=FD=20 N dan FB = FC = 40 N. Moment gaya total pada titik perpo- tongan diagonal bangun tersebut adalah…. A. 2,16 Nm B. 2,20 Nm C. 2,80 Nm D. 3,36 Nm E. 4,00 Nm
  • 25. 8. Momen Inertia Prediksi Soal 1) Momen inertia partikel Jika sistem diputar pada salah satu sumbunya , maka momen inersia sistem adalah.... m 3 m m a a 3 a X Y 2 2 2 2 1 1 2 1 r m r m I I I I      
  • 26. Prediksi Soal 2) Momen inertia benda pejal diketahui massa batang M dan panjang batang L  diputar di salah satu ujungnya  diputar di sembarang titik tegak lurus batang A B A B r2 r1 2 3 1 ML I    3 2 3 1 3 r r I L M  
  • 27.  Letak koordinat titik berat bidang terhadap titik A (0,0) adalah …. 6 cm 6 cm √13 cm 9 cm A B C D E Ringkasan Materi:  Letak titik berat persegi panjang pada titik potong kedua diagonal sudut  Letak titik berat bangun segitiga sama sisi pada jarak sepertiga tinggi bangun  Untuk gabungan beberapa benda homogen harus dipisahkan terlebih dahulu, kemudian ditentukan letak titik berat dan luas masing-masing selanjutnya dihitung letak titik berat gabungan.  Bila benda bukan bidang tetapi berupa garis atau memiliki volume, maka luas(A) diganti dengan panjang benda(L) atau volume benda (V) Benda X Y Luas A 1 X1= .... Y1= .... A1= .... 2 X2= .... Y2= .... A2= .... dst .... .... .... 2 1 2 2 1 1 0 .... A A X A X A X     2 1 2 2 1 1 0 .... A A Y A Y A Y    
  • 28. Prediksi soal • Perhatikan gambar-gambar berikut lalu tentukan letak koordinat titik beratnya 6 6 6 15 7,5 0 15 7,5 x (cm) y (cm)
  • 29. Perhatikan gambar berikut. Agar dikehendaki kedua benda setimbang, maka didekat benda yang 2 kg perlu ditambahkan benda lain yang massa dan posisi benda masing-masing … A. 19 kg dan 1 m sebelah kiri penumpu B. 12 kg dan 1,1 m sebelah kiri penumpu C. 10 kg dan 0,5 m kanan penumpu D. 10 kg dan 1,0 m kanan penumpu E. 5,0 kg dan 1,3 m kiri penumpu Kesetimbangan Benda Tegar  Syarat setimbang: sistem tidak bergeser dan tidak berputar   nol F     nol   Gambar di bawah menunjukkan 4 benda yang berada dalam keadaan seimbang. Keseimbangan yang stabil ditunjukkan oleh gambar …
  • 30. Tangga dengan massa 40 kg disandarkan pada dinding seperti gambar.  Agar batang berada dalam posisi stabil, nilai koefisien gesek minimum adalah ....  Tinggi maksimum seorang anak bermassa 40 kg menaiki tangga sebelum tergelincir adalah… . 3 m 5 m α Sebuah batang homogen AC dengan panjang 4 m dan massanya 50 kg. Pada ujung C digantungkan beban yang massanya 20 kg. Batang ditahan oleh tali T sehingga sistem seimbang. Jika jarak BC 1 m, maka hitunglah tegangan tali T!
  • 31. Batang homogen AB dengan berat 10 N. Pada ujung batang digantung beban seperti pada gambar di bawah. Jika sistem di atas dalam keadaan seimbang, maka besar tegangan tali adalah …. A. 10,0 N B. 16,7 N C. 25,0 N D. 31,3 N E. 41,7 N
  • 32. 1) Hukum Archimedes Ringkasan Materi suatu benda yang dicelupkan sebagian atau seluruhnya ke dalam zat cair mengalami gaya ke atas yang besarnya sama dengan berat zat cair yang dipindahkan oleh benda tersebut :  A (terapung) & B (melayang), besar gaya angkat = berat benda di udara :  C (tenggelam), besar gaya angkat = berat benda di udara – berat benda di dalam air :  Perbandingan gaya angkat dengan gaya berat di udara : A B C benda benda air air benda air air A V V g m gV W F       air A W W F   benda air A W F   
  • 33. Prediksi Soal : 1)Hukum Archimedes massa jenis fluida Sebuah balok dimasukkan ke dalam wadah A dan wadah B yang berisi zat cair yang berbeda dan hasilnya seperti gambar diatas. Maka perbandingan massa jenis zat cair dalam wadah A, wadah B dan wadah C adalah… A B B benda A benda B A h h h h h h   / /   B C C benda B benda C B h h h h h h   / /   A C C benda A benda C A h h h h h h   / /   ; & A B C h h h adalahbagianbenda yangtercelupcairan
  • 34. Contoh Soal: 1. Sebuah kubus mempunyai rusuk 10 cm, dimasukkan ke dalam minyak bermassa jenis 800 kg.m-3, ternyata bagian kubus yang muncul di permukaan air adalah setinggi 4 cm seperti gambar dibawah. Jika g = 10 m.s-2, maka besarnya gaya keatas yang bekerja pada kubus adalah … 2. Sebuah balok massa jenisnya 2.500 kg.m-3 dan ketika di udara beratnya 25 Newton. Tentukan berat balok di dalam air jika massa jenis air 1000 kg.m-3 dan percepatan gravitasi bumi 10 m.s-2 ! 3. Peti berukuran 30 cm x 40 cm x 50 cm dengan berat 1.000 N hendak diangkat dari dasar laut (massa jenis air laut =1,1 x 103 kg.m3 dan g = 10 ms-2) . Gaya minimal yang dibutuhkan untuk mengangkat peti tersebut adalah …
  • 35. Peti berupa kotak berukuran 100 cm x 150 cm x 200 cm yang berada di dasar laut hendak diangkat ke permukaaan laut dengaan cara menariknya dengan rantai. Bila massa jenis rata- rata peti = 2000 kg.m-3 dan massa jenis air laut = 1000 kg.m-3 , maka besar gaya angkat minimal yang dibutuhkan untuk mengangkat peti tersebut adalah …. (abaikan massa rantai). A. 2,0 kN B. 3,0 kN C. 6,0 kN D. 9,0 kN E. 18,0 kN
  • 36. Pernyataan terkait dengan gambar 1) gaya F pada pipa kecil, dongkrak dapat mengangkat benda yang beratnya 4F pada pipa besar 2) gaya F pada pipa kecil, dongkrak dapat mengangkat benda yang beratnya 4W pada pipa besar 3) tekanan yang ditimbulkan gaya F pada pipa kecil sama dengan tekanan yang ditimbulkan gaya berat pada pipa besar 4) tekanan yang ditimbulkan gaya F pada lebih besar dari tekanan yang ditimbulkan gaya berat pada pipa besar Penyataan yang benar adalah … Dongkrak hidrolik pada gambar memiliki perbandingan jari-jari penampang pipa kecil dan penampang pipa besar adalah 1 : 2 Dongkrak hidrolik digunakan untuk mengangkat beban seperti gambar di samping. Jika jari – jari pada pipa kecil adalah 2 cm dan jari – jari pipa besar adalah 18 cm, maka besar gaya minimal yang diperlukan untuk mengangkat beban 81 kg adalah….(g=10 m.s-2)
  • 37. Perhatikan tangki bocor berikut ! Jika air dalam tangki berisi penuh, maka kecepatan pancaran air dan jarak pancaran terjauh adalah.... Ringkasan Materi Azas Torricelli  Laju pancaran air:  Waktu yang diperlukan sampai tanah:  Jarak pancaran: h1 : tinggi permukaan air terhadap lubang bocoran h1 : tinggi lubang bocoran terhadap tanah x h h2 v h1 1 2gh v  g h t 2 2  2 1 2 xh h x  3) Hukum Bernouli Ditanyakan R?
  • 38. Penerapan hukum Bernoulli pada gaya angkat pesawat terbang. Bentuk penampang sayap bagian belakang lebih tajam dan bagian atas melengkung menyebabkan aliran udara di bagian atas lebih besar dari bagian bawah (v2 > v1). Dari persamaan Bernoulli : P1 + ½ .v1 2 + gh1 = P2 + ½ .v2 2 + gh2 Karena Ketinggian kedua sayap dapat dianggap sama (h1=h2), maka : P1 + ½ .v1 2 = P2 + ½ .v2 2 P1 – P2 = ½  (v2 2 – v1 2) karena F=P x A Maka : F1 – F2 = ½  A(v2 2 – v1 2) dengan  = massa jenis udara (kg/m3) Supaya pesawat dapat terangkat, gaya angkat harus lebih besar daripada berat pesawat (F1 - F2 > mg). Saat pesawat dalam keadaan melayang di udara), kelajuan pesawat harus diatur sedemikian rupa sehingga gaya angkat sama dengan gaya berat pesawat (F1 - F2 = mg). 1. Gaya angkat pada pesawat terbang dapat terjadi karena... 1) Tekanan udara di atas sayap lebih besar dari pada di bawah sayap 2) Kecepatan udara di atas sayap lebih besar dari pada di bawah sayap 3) Tekanan udara di atas sayap lebih kecil dari pada di bawah sayap 4) Kecepatan udara di atas sayap lebih kecil dari pada di bawah sayap 2. Sebuah pesawat dilengkapi dengan dua buah sayap masing-masing seluas 40 m2 (anggap kerapatan udara adalah 1,2 kg.m- 3). Jika kelajuan aliran udara di atas sayap adalah 250 m.s-1 dan kelajuan udara di bawah sayap adalah 200 m.s-1, maka besar gaya angkat pada pesawat tersebut adalah … .
  • 39. 1. Sebuah benda bermassa 2 kg mula-mula diam karena pengaruh gaya luar setelah menempuh jarak 5 m kecepatannya menjadi 6 ms-1. Usaha yang dilakukan oleh gaya luar adalah …. W=ΔEk =½m(vt 2-v0 2) 2. Benda bermassa m dari keadaan diam mendapat gaya F sehingga mengalami percepatan a. Ditanyakan usaha yang dilakukan setelah t sekon atau setelah perpindah sejauh s 3. Sebuah peluru dengan massa 200 gram ditembak vertikal keatas dari permukaan tanah dengan kecepatan 60 m.s-1. Jika g = 10 m.s-2, maka energi kinetik peluru pada ketinggian 40 m adalah…. 4. Bola yang massanya 1 kg dilepas dari titik A menempuh lintasan seperti gambar. Jika lintasan AB adalah seperempat lingkaran licin dengan jari-jari 75 cm, maka perbandingan kecepatan bola di titik B dan C adalah ....
  • 40. Sebuah balok bermassa 2 kg ditarik oleh gaya F yang membentuk sudut tertentu terhadap bidang datar sehingga berpindah seperti gambar di bawah. Urutan usaha yang dari yang paling kecil ke besar yang dilakukan oleh gaya F untuk menarik balok adalah … No m (kg) F (N)  (o) t (s) 1 1 5 60 10 2 2 5 37 10 3 1 6 53 5 4 2 4 37 10 5 2 5 53 12
  • 41. 1). Sebuah benda yang massanya 4 kg dijatuhkan dari ketinggian 200 m diatas permukaan tanah tanpa kecepatan awal. Jika percepatan gravitasi di tempat tersebut 10 m.s-2, maka usaha yang dilakukan oleh benda saat mencapai ketinggian 60 m diatas permukaan tanah adalah.... 2) Contoh lain 3 m β F 1 m 300 A B   .... 1 2    h h mg W   .... sin   s mg W 
  • 42. 1) Sebuah benda bermassa sama 4 kg, jatuh bebas dari ketinggian 10 meter diatas permukaan tanah. Jika percepatan gravitasi 10 m.s-2 , maka energi kinetik benda saat benda mencapai ketinggian 1 m diatas tanah sebesar.... 2) Sebuah kelereng meluncur tanpa gesekan dari titik A ke titik B seperti gambar (g = 10 ms-2) Jika kelereng (m=10 gram) mula-mula diam, maka energi kinetik / laju di B adalah …..   .... 1 2    h h mg Ek   .... ) ( 2 ....       A B A B h h g v h h mg Ek Ditanyakan v ?
  • 43. Balok bermassa m meluncur dari keadaan diam sepanjang bidang miring licin seperti gambar di bawah ini. Perbandingan kecepatan balok saat melintasi titik Q dan S adalah …. A. √1 : √2 B. √1 : √ 4 C. √2 : √5 D. √4 : √1 E. √5 : √2
  • 44. Perhatikan gambar benda yang diluncurkan dan menumbuk benda lain yang sedang diam. Benda m1 dan m2 mula mula diam. Jika setelah m1 meluncur terjadi tumbukkan lenting sempurna antara m1dan m2, percepatan gravitasi 10 m.s-2 dan kecepatan benda m1 setelah tumbukkan adalah 1 m.s-1 maka kecepatan benda m2 setelah tumbukkan adalah … Bola P bermassa 2 kg meluncur dari ketinggian h kemudian menumbuk benda B yang diam seperti gambar di bawah. Jika massa bola Q = 4 kg (g = 10 m.s-2) dan setelah bertumbukkan bola P berhenti, berapakah besar energi kinetik bola Q akibat tumbukkan tersebut? Q h = 5 m P
  • 45. Silinder pejal dengan jari-jari 5 cm bermassa 0,25 kg bertranslasi dengan kelajuan linear 4 m/s. Tentukan energi kinetik silinder jika selain bertranslasi silinder juga berotasi! Pembahasan Data dari soal: m = 0,25 kg r = 5 cm = 0,05 m v = 4 m/s Ek =..... Jawab: Sebuah bola pejal terbuat dari besi menggelinding pada lantai dasar dengan laju 15 m.s-1. Jika massa bola 2 kg dan diameternya 40 cm, maka energi kinetik total bola adalah… . A. 90 J B. 225 J C. 315 J D. 400 J E. 525 J
  • 46. Bola 1 dan bola 2 di putar masing-masing dengan kecepatan ω1 dan ω2 seperti pada gambar di bawah. Bola 1 Bola 2 Perbandingan jari-jari bola 1 dan bola 2 = 2 : 3, sedangkan perbandingan massa bola 1 dan bola 2 = 1 : 3. Bila energi kinetik rotasi kedua bola sama, maka perbandingkan kecepatan sudut bola 1 dan 2 adalah ….
  • 47. Sebuah piringan bermassa 2 kg berbentuk silinder pejal (I = ½ mR2) dengan jari-jari 0,2 m mula-mula berputar di atas meja dengan kelajuan sudut 5 rad.s-1. Bila di atas piringan diletakkan cincin bermassa 0,1 kg dan jari-jari 0,2 meter (I = mR2) dengan poros putar cincin tepat di atas pusat piringan, maka piringan dan cincin akan bersama-sama berputar dengan kecepatan sudut .… A. 0,22 rad.s-1 B. 2,00 rad.s-1 C. 2,20 rad.s-1 D. 4,50 rad.s-1 E. 44,00 rad.s-1
  • 48. Sebuah bola bermassa 400 gram yang sedang bergerak mendatar menumbuk tembok dan dipantulkan seperti gambar. Besar Impuls yang dialami bola adalah … . Kuprett solusion I = m (v2 – v1) I =…… (…… – ……) I = …. Nm Contoh penerapan seperti ditunjukkan pada gambar di atas bertujuan untuk .... A. memperbesar gaya tumbukkan dengan cara memperbesar waktu tumbukkan B. meredam gaya tumbukkan dengan cara memperbesar waktu tumbukkan C. memperbesar gaya tumbukkan dengan cara mempersingkat waktu tumbukkan D. meredam gaya tumbukkan dengan cara memperkecil waktu tumbukkan E. memperbesar impuls tumbukkan dengan cara memperbesar waktu tumbukkan
  • 49. 2a). Sebuah benda yang jatuh bebas menumbuk lantai seperti gambar, impuls yang diterima benda adalah …. N.s Pembahasan: Tip: hitung dahulu besar v1 dan v2 dengan menggunakan rumus: 2b). Bola bermassa 0,2 kg jatuh bebas dari ketinggian 1,8 m di atas lantai dan memantul kembali dengan kecepatan 2 m.s-1 dengan arah berlawanan. Jika percepatan gravitasi 10 m.s-2 , maka impuls yang terjadi adalah.... gh vt 2  -v1 h 1 = 5 m h 2 =0,8 m +v1 ’ m = 2 kg   .... 1 2      v v m p I
  • 50. Ringkasan Materi Tumbukan Pada peristiwa tumbukan antara dua benda berlaku hukum kekekalan momentum yaitu jumlah momentum kedua benda sesaat sebelum dan sesudah tumbukan adalah sama pawal = pakhir mava +mbvb = mava 1 +mbvb 1 Untuk menentukan kecepatan sesaat masing-masing benda sesudah tumbukan:  Langkah pertama anggap kecepatan kedua benda sesaat sesudah tumbukan adalah :  Langkah kedua hitung kecepatan masing benda: Dimana e : koefiesien restitusi yang besarnya adalah 0 < e < 1  e=0 , kedua benda bertumbukan tidak lenting sama sekali sehingga keduanya menyatu, maka:  e=1, kedua benda bertumbukan lenting sempurna, maka: 0 < e < 1, kedua benda bertumbukan lenting sebagian gunakan rumus pada langkah kedua A B vA VB B A B B A A m m v m v m x        B B A A v x e x v v x e x v       1 1 B B A A v x v v x v     2 2 1 1 15. Tumbukan x v v B A   1 1
  • 51. Tumbukan Lenting Dua Dimensi Massa masing-masing partikel adalah mA dan mB. Dengan menerapkan hukum kekekalan momentum pada arah sumbu x dan sumbu y, diperoleh:  pada sumbu x.  pada sumbu Y (Voy = nol)  perbandingan sinus kedua sudut :
  • 52. Sebuah bola dilemparkan ke tembok untuk kemudian memantul seperti tampak pada gambar di bawah. Jika tumbukan antara bola dan dinding bersifat lenting sempurna, maka besar perubahan momentum bola adalah …. A. nol B. 2,0 kg m.s-1 C. 4,0 kg m.s-1 D. 8,0 kg m.s-1 E. 16 kg m.s-1
  • 53.  Bola A dan B berada pada ketinggian yang sama seperti gambar, masing-masing massanya 12 kg dan 4 kg dilepas- kan pada ketinggian bidang lengkung, sehingga bola A dan bola B bertumbukan di O keduanya menyatu bergerak bersama-sama maka kecepatan kedua bola adalah .... Tip: hitung dahulu besar v1 dan v2 dengan menggunakan rumus: gr vt 2  ... .... 2 .... 2 1 1           x v v m m v m v m x gh v gh v B A B A B B A A B B A A Kuprett solusion :
  • 54.  Bola A dan B masing-masing bermassa 1 kg dan 2 kg secara bersamaan dilepaskan hingga meluncur di atas bidang yang licin seperti gambar. Kedua bola bertumbukan lenting sempurna di bidang datar. Kecepatan bola B sesaat setelah bertumbukan adalah .... A B 5 m 3,2 m ... 2 ... 2 ... 2 ... 2 1 1              B B A A B A B B A A B B A A v x v v x v m m v m v m x gh v gh v Kuprett solusion :
  • 55.  Sebuah gelas kaca memiliki kapasitas 400 cm3, penuh berisi air pada suhu 200C. Jika gelas beserta air tersebut dipanaskan hingga suhunya menjadi 700C (αkaca = 9 x 10-6 0C-1 dan air = 2,1 x 10-4 0C-1), maka banyak air yang tumpah adalah :  Sebuah wadah 95 % dari kapasitasnya diisi cairan yang bersuhu 00C. Wadah terbuat dari bahan dengan koefisien muai volume 8,0 x 10-5/ 0C. Wadah yang telah berisi cairan tersebut dipanaskan hingga tepat pada suhu 100 0C ternyata cairan mulai tumpah. Koefisien muai volume cairan tersebut adalah … ..... ) 3 ( 0      t V V   ..... 3 0        t V V
  • 56.  Logam yang massanya 200 gram bersuhu 900C dimasukkan ke dalam150 gram air bersuhu 350C. Kalor jenis logam dan air masing-masing 700 J/kgK dan 4200 J/kgK. Setelah terjadi keseimbangan termal suhu air menjadi....  Sepotong logam 10 gram bersuhu 100 oC dimasukkan ke dalam 50 gram zat cair yang bersuhu 25 oC, akhirnya kedua zat mencapai keseimbangan pada suhu 50 oC. Jika kalor jenis zat cair 20 kal.g-1.oC-1, maka kalor jenis zat tersebut adalah ... .         c air c air air t t m t t c m air c air air c air c t t c m t t c m Q Q        logam logam logam logam logam logam logam
  • 57. Es bersuhu 0oC,dimasukkan ke dalam air bermassa 340 gram suhu 20oC yang ditempatkan pada bejana khusus. Anggap bejana tidak menyerap/melepaskan kalor. Jika kalor lebur es Les = 80 kal g-1dan kalor jenis air cair= 1 kal g-1 oC-1 dan semua es mencair dengan suhu kesetimbangan termal = 5oC, maka besar massa es adalah .… A. 60 gram B. 68 gram C. 75 gram D. 80 gram E. 170 gram
  • 58. Batu es bermassa 400 gram bersuhu -10 oC dipanaskan dengan proses seperti ditunjukkan oleh grafik hubungan antara jumlah kalor (Q) dan suhu (t) di bawah. Jika kalor jenis es = 0,5 kal.gram-1.0C-1, kalor jenis air = 1 kal.gram-1.0C-1 dan kalor lebur es 80 kal/gram, berapakah kalor total untuk mengubah batu es menjadi uap seluruhnya?
  • 59. Laju Perpindahan Kalor  Konduksi:  Konveksi:  Radiasi: Pertukaran kalor antara dua benda yang suhunya berbeda memiliki laju perpindahan kalor yang sama. Prediksi Soal : Sebuah balok logam yang panjangnya L dipotong menjadi tiga bagian yang sama, kemudian dicat dengan warna yang berbeda seperti gambar. Bila ketiga logam dijemur dalam waktu yang sama, bagaimana kenaikan suhu yang terjadi? d T A k H   T A h H   . . 4 T e W   A B C
  • 60. Perhatikan tabel kalor jenis dari beberapa logam berikut. Jika secara bersamaan keempat logam tersebut di panaskan dalam waktu yang sama, maka urutan panas benda dari kurang panas ke lebih panas adalah … Dua logam A dan B memiliki luas penampang dan panjang sama disambung seperti pada gambar. Koefisien konduksi termal logam A = 2 kali koefisien konduksi termal logam B, maka suhu sambungan kedua logam tersebut adalah .... A. 20 oC B. 30 oC C. 40 oC D. 60 oC E. 80 oC
  • 61. Dua batang P dan Q dengan ukuran yang sama tetapi jenis logam berbeda dilekatkan seperti pada gambar di bawah. Jika koefisien konduksi termal P adalah dua kali koefisien konduksi termal Q, maka besar suhu di titik T adalah …. A. 600 oC B. 200 oC C. 90 oC D. 60 oC E. 45 oC
  • 62. Sifat-sifat gas ideal 1) Gas terdiri atas partikel-partikel padat kecil (mempunyai masa yang sama) yg bergerak dengan kecepatan tetap dan dengan arah sembarang. 2) Masing-masing partikel bergerak dalam garis lurus, gerakan partikel hanya dipengaruhi oleh tumbukan antara masing-masing partikel atau antara partikel dan dinding. Gaya tarik-menarik antarpartikel sangat kecil sekali dan dianggap tidak ada (diabaikan). 3) Tumbukan antara masing-masing partikel atau antara partikel dengan dinding adalah tumbukan lenting sempurna. 4) Waktu terjadinya tumbukan antarpartikel atau antara partikel dengan dinding sangat singkat dan bisa diabaikan. 5) Ukuran volume partikel sangat kecil dibandingkan ukuran volume ruang tempat partikel tersebut bergerak. Berlaku hukum Newton tentang gerak.
  • 63. Persamaan gas ideal (mono atomik)  Hukum Boyle-Gay lussac Prediksi Soal: NEk PV kT Ek NkT nRT PV Nk nR c c T PV 3 2 2 3          Tekanan gas ideal dalam ruang tertutup terhadap dinding tabung dirumuskan : ;[p = tekanan (Pa); N = jumlah molekul (partikel) gas; V = volume gas dan Ek adalah energi kinetik rata-rata molekul (J)]. Berdasarkan persamaan ini pernyataan yang benar adalah .... 1. tekanan gas terhadap dinding bergantung pada energi kinetik rata-rata molekul 2. energi gas bergantung pada tekanan yang ditimbulkan molekul terhadap dinding 3. suhu gas dalam tabung akan berubah jika tekanan gas berubah 4. jumlah molekul gas berkurang maka volume energi kinetik molekul akan berkurang 5. jika volume gas bertambah maka tekanan akan berkurang Ek P V N 3 2 
  • 64. Gas dengan volume 7 liter pada suhu 27°C dan tekanan 2 atm dimampatkan hingga tekanan dan suhunya mencapai 6 atm dan suhu 42°C. Volume gas sekarang adalah …. A. 0,61 liter B. 2,45 liter C. 6,10 liter D. 14,70 liter E. 24,50 liter
  • 65.  Hukum Melde (besaran gelombang) 1) Jika panjang dawai yang bergetar (L) 120 cm dan frekuensi vibrator 50 Hz, maka cepat rambat gelombang pada dawai adalah … . 2) Jika cepat rambat gelombang pada dawai 20 m/s dan frekuensi vibrator 50 Hz, maka letak perut ketiga dan simpul kelima diukur dari ujung pantul adalah … .                      2 1 4 1 2 ... n Xs n Xp f v n n ... . ... 2 3      f v L   
  • 66.  Persamaan gelombang Sebuah gelombang berjalan memenuhi persamaan y = 0,2 sin 0,4 π (60t – x), x dan y dalam cm dan t dalam sekon. Tentukan: 1) amplitudo gelombang 0,2 cm 2) periode gelombang, 12 s 3) panjang gelombang, 5 cm 4) cepat rambat gelombang 60 cm/s Pernyataan di atas yang benar adalah ….                   x f t A y kx t A y 2 2 sin sin .... ..... 2 .... 2 1 .... 2 ....             k f T v cm k sekon f T Hz f cm A          Sebuah gelombang merambat sepanjang tali dinyatakan dengan persamaan , , dalam cm. Beda fase antara dua titik yang berjarak 5 cm dan 10 cm dari asal getaran adalah… .
  • 67. Perhatikan gambar gelombang yang merambat pada tali di bawah ini. Beda fase antara titik b dan d serta titik h dan i adalah …. A. ¼ dan ½ B. ¼ dan ¾ C. ½ dan ¼ D. ½ dan ½ E. ½ dan ¾
  • 68. Suatu gelombang stasioner memenuhi persamaan y = 0,1 cos (4x) sin (20t) dengan x, y dalam meter dan t dalam sekon. Jika panjang tali 2 m maka letak simpul ke lima dari titik pantul adalah … Ujung seutas tali diikat mati pada sebuah tiang, sedangkan ujung tali yang lain digetarkan bolak balik ke atas dan ke bawah sehingga terbentuk gelombang stasioner dengan persamaan: y = 0,5 sin (5πx) cos π(10t–4) m. Jarak x terdekat dari ujung pantul pada saat amplitudo gelombang stasionernya = 25 cm adalah …
  • 69.  Dua pipa organa terbuka A dan pipa organa tertutup B ditiup bersama- sama. Pipa A menghasilkan nada dasar yang sama tinggi dengan nada atas kedua pipa B. Jika panjang pipa organa A 10 cm, maka panjang pipa organa B adalah ... .  Sebuah pipa organa terbuka mengha- silkan nada atas kedua dengan frekuensi x Hz, sedangkan pipa organa tertutup B menghasilkan nada atas ketiga dengan frekuensi y Hz. Bila panjang, suhu dan jenis gas dalam kedua pipa organa sama, perbandingan y dengan x adalah …
  • 70. Sebuah pipa organa terbuka ditiup dengan keras sehingga menghasilkan nada atas ketiga dengan frekuensi 1700 Hz. jika cepat rambat bunyi di udara 340 m.s-1, maka panjang pipa organa tersebut adalah …. A. 50 cm B. 40 cm C. 30 cm D. 25 cm E. 15 cm
  • 71.  Efek Doppler Rumus Efek Doppler:  Bila hembusan angin dapat diabaikan:  Bila hembusan angin tidak dapat diabaikan: fp: frekuensi bunyi diterima pendengar(Hz) fs : frekuensisumber bunyi(Hz) v : cepat rambat bunyi di udara(m/s) vp: kecepatan pendengar(m/s), bertanda(+) bila pendengar(P) menuju sumber bunyi(S) vs : kecepatan sumber bunyi(m/s), bertanda (+) bila sumber bunyi(S) menjauhi pendengar(P) va : kecepatan angin(m/s), bertanda (+) bila bertiup dari arah sumber bunyi(S) menuju pendengar(P) s s p p f v v v v f            s a s a p p f v v v v v v f              P S1 s2 S P1 P2 S P S P P P P P v v v v f f    2 1 S S P P v v v v f f    2 1       S P S P P P v v v v v v v v f f      2 1
  • 72. Mobil pemadam kebakaran sedang bergerak dengan laju 20 m.s−1 sambil membunyikan sirine pada frekuensi 400 Hz (cepat rambat bunyi 300 m.s−1). Jika mobil pemadam kebakaran bergerak menjauhi Amri yang berdiri di tepi jalan, maka perbedaan frekuensi sirine yang didengar oleh Amri dengan frekuensi sumbernya adalah .... A. 384 Hz B. 375 Hz C. 75 Hz D. 25 Hz E. 16 Hz
  • 73. Prediksi soal Sebuah mobil ambulan bergerak dengan kecepatan 36 km.jam-1 melintas di depan pendengar yang berdiri di tepi jalan sambil membunyikan sirine dengan frekuensi 660 Hz. Jika cepat rambat bunyi di udara 340 m.s-1 maka perbandingan frekuensi yang didengar pada saat mobil ambulan menjauhi dan mende- kati pendengar adalah.... Pembahasan: Kereta A dan B bergerak saling mendekati dengan kecepatan masing-masing 20 ms-1 dan 15 ms-1. Masinis kereta B membu- nyikan peluit dengan frekuensi 660 Hz, sedangkan kereta A membunyikan pluit dengan frekuensi 735 Hz. Jika cepat rambat bunyi di udara 345 ms-1, maka orang dalam kereta A akan mendengar layangan bunyi dengan frekuensi …
  • 74.  Intensitas bunyi Ringkasan Materi:  Intensitas Bunyi:  Perbandingan intensitas bunyi antara 2 titik berjarak r1 dan r2 dari sumber bunyi:  Amplitudo : Taraf intensitas: Satu sumber bunyi : n sumber bunyi : TIn = TI1 + 10 log n antara 2 titik berjarak r1 dan r2 dari sumber bunyi: A P I  2 1 2 2 1 I r I r        1 2 1 2 20log r TI TI r   Prediksi soal 1) Perhatikan gambar Jika S adalah sumber bunyi yang merambat ke segala arah dengan daya 100 watt, maka perbandingan intensitas bunyi pada titik A, B dan C adalah …. Kuprett solusion : S A B C 4 m 9 m 12 m   9 : 81 : 16 : : 36 4 9 12 1 : 4 1 : 9 1 : : 1 : 1 : 1 : : 2 2 2 2 2 2          C B A C B A C B A C B A I I I x x I I I r r r I I I 2) Titik P, Q, dan R mempunyai jarak masing – masing 2 m, 6 m dan 3 m dari alarm yang sedang berbunyi. Perbandingan intensitas bunyi di titik P, Q dan R adalah....   .... : .... : .... : : ..... .... .... .... 1 : .... 1 : .... 1 : : 1 : 1 : 1 : : 2 2 2 2 2 2          C B A C B A R Q P R Q P I I I x x I I I r r r I I I 0 log 10 I I TI  1 2 2 1 A r A r 
  • 75. Bunyi yang didengar pada jarak 3 m dari sumbernya memiliki intensitas 10-4 W.m2. Taraf intensitas bunyi tersebut saat didengar pada jarak 300 m dari sumber bunyi adalah …. (Io = 10-12 W.m2) A. 20 dB B. 30 dB C. 40 dB D. 60 dB E. 90 dB
  • 76. Ringkasan Materi Mikroskop Mata tak berakomodasi Ciri-ciri: Lensa Okuler menghasilkan dua berkas sinar sejajar maka mata tak berakomodasi Rumus: Mata berakomodasi maksimum Ciri-ciri: Lensa Okuler menghasilkan dua berkas sinar yang melebar maka mata berakomodasi maksimum Rumus: Ringkasan Materi Teropong Teropong Bintang Ciri-ciri: Lensa Okuler menghasilkan berkas sinar sejajar karena benda yang diamati jaraknya jauh sehingga mata tak berakomodasi Rumus: obyektif okuler S 1 oby=foby S oku= foku oku oby oby F PP x S S M 1  1 1   oku oby oby F PP x S S M oku oby oku oby f f d f f M    Ringkasan Materi LUP LUP (kaca pembesar) pada mikroskop dan teropong berfungsi sebagai Lensa Okuler Rumus:  Kekuatan lensa :  mata berakomodasi maksimum :  mata tak berakomodasi : 1   f PP M     cm m f f P 100 1     " " 100 cm dalam PP satuan bila P PP f PP M   
  • 77. 1) Sebuah mikroskop jarak fokus okulernya 2,5 cm dan jarak fokus objektifnya 0,9 cm, digunakan oleh orang bermata normal (Sn = 25 cm) tanpa berakomodasi dan ternyata perbesarannya 90 kali . Berarti jarak objektif terhadap lensa objektifnya adalah …. Kuprett solusion : .... 5 , 2 25 9 , 0 9 , 0 90                                    ob ob ok ob ob ob s s f PP f S f M
  • 78. Prediksi soal Perhatikan pembentukan bayangan pada teropong bintang berikut. Perbesaran bayangan yang dihasilkan adalah.... Kuprett solusion : Gambar pembentukan bayangan pada teropong bintang berikut ini : Agar bayangan akhir benda dapat diamati pada layar, maka lensa okuler digeser sejauh… . Kuprett solusion : ....     oku oby oku oby f f M f d f
  • 79. Prediksi soal Sebuah teropong bintang memiliki lensa obyektif dengan jarak fokus 100 cm dan lensa okuler dengan jarak fokus 5 cm. Teropong itu digunakan untuk mengamati benda langit dengan mata tak berakomo- dasi. Berapa cm lensa okuler harus digeser agar bayangan dapat ditangkap dengan jelas pada sebuah layar yang dipasang pada jarak 10 cm di belakang okuler dan kemana arah pergeserannya ? Pembahasan: Sebuah teropong bintang memiliki lensa objektif dengan jarak fokus 5 cm dan jarak focus lensa okulernya cm. Teropong itu digunakan untuk mengamati benda langit dengan mata tak berakomodasi. Agar bayangan dapat ditangkap dengan jelas pada sebuah layar yang dipasang pada jarak 10 cm di belakang okuler, maka besar jarak dan arah lensa okuler harus digeser adalah …
  • 80. Teropong itu digunakan untuk mengamati benda langit dengan mata tak berakomodasi, jalannya sinar seperti gambar Berapa cm lensa okuler harus digeser agar bayangan dapat ditangkap dengan jelas pada sebuah layar yang dipasang pada jarak 10 cm di belakang okuler dan kemana arah pergeserannya ? A. 3 cm menjauhi lensa objektif. B. 3 cm mendekati lensa objektif. C. 5 cm menjauhi lensa objektif. D. 5 cm mendekati lensa objektif. E. 10 cm menjauhi lensa objektif.
  • 81. 3) Sebuah lensa memiliki kekuatan 20 dioptri digunakan sebagai lup. Jika mata normal menggunakan lup tersebut dengan berakomodasi maksimum, maka perbesaran anguler lup adalah … Kuprett solusion :       .... 1 100 1 100 1        P PP f PP M f f P cm m
  • 82. Prediksi soal Seberkas sinar monokromatis dengan panjang gelombang 6.000 Å (1 Å = 10-10 m) melewati celah tunggal menghasilkan pola difraksi orde terang pertama seperti pada gambar. Lebar celahnya sebesar .... Kuprett solusion : sinar 300 .... sin     n d
  • 83. Prediksi soal 1) Pada percobaan Young digunakan 2 celah sempit yang berjarak 2 mm satu sama lain dan layar yang dipasang 1 m dari celah tersebut. Jika dihasilkan terang kedua pada jarak 0.5 mm dari terang pusat, maka panjang gelombang cahaya yang digunakan adalah… Rumus yang diperlukan:  Interfernsi maksimum : d sin  = m  jarak pita gelap dan terang berdekatan :  panjang gelombang cahaya yang digunakan : 2) Pada percobaan young (celah ganda), jika jarak antara dua celahnya dijadikan dua kali semula, maka jarak antara 2 garis gelap yang berurutan menjadi A. 4 kali semula B. 2 kali semula C. ½ kali semula D. ¼ kali semula E. Tetap .... sin    mL dy m d   d L y 2  
  • 84. 3) Seberkas cahaya monokromatis jatuh tegak lurus mengenai dua buah celah sempit hinga terjadi interferensi. Jika jarak antara dua celah sempit adalah 4 x 10-2 cm, jarak antara layar dan celah 0,4 cm serta garis terang ketiga yang terbentuk berjarak 5 x 10-2 cm dari terang pusat, maka panjang gelombang cahaya yang digunakan adalah.... A. 1.667 Å B. 2.873 Å C. 3.870 Å D. 4.652 Å E. 5.400 Å Pembahasan:
  • 85. Pada percobaan young (celah ganda), jika jarak antara dua celahnya dijadikan dua kali semula, maka jarak antara 2 garis gelap yang berurutan menjadi A. 4 kali semula B. 2 kali semula C. ½ kali semula D. ¼ kali semula E. Tetap
  • 86.  Kotak nomor 1 sampai dengan 5 pada gambar di bawah mewakili jenis-jenis gelombang elektromagnetik tertentu. menyebutkan nama gelombang dan manfaatnya/bahayanya !
  • 87. Perhatikan gambar di bawah ini! Gambar diatas menunjukkan spektrum gelombang elektromagnetik. Pernyataan yang benar berdasarkan gambar di atas adalah .... A. P merupakan sinar X yang berfungsi untuk mengetahui bagian dalam tubuh manusia B. Q merupakan gelombang mikro yang berfungsi untuk pengamanan bandara C. R merupakan inframerah untuk penerangan D. P merupakan gelombang mikro untuk memanaskan makanan E. Q merupakan sinar X untuk fisioterapi
  • 88. Predik Soal 1) Grafik berikut menggambarkan hasil percobaan dari sebuah pegas yang digantungi beban. Besarnya x adalah.... 2) Grafik hubungan gaya (F) terhadap pertambahan panjang (x) dari dua pegas A dan pegas B seperti pada gambar di atas, maka ........ F (kg.m.s-1) ∆X (cm) A B C 6 15 25 5 6  tan 1       k garis kemiringan k x F k .... 5 , 1 2 6 2 2 2 2 1 1      x x x F x F k
  • 89. 10. Grafik berikut menggambarkan hasil percobaan dari sebuah pegas yang digantungi beban. Besarnya x adalah.... A. 1,5 cm B. 2,0 cm C. 4,0 cm D. 6,0 cm E. 8,0 cm Pembahasan: .... 5 , 1 2 6 2 2 2 2 1 1      x x x F x F k
  • 90. 3) Perhatikan grafik hubungan F dengan pertambahan panjang pegas ∆L Pegas yang memiliki konstanta paling besar ditunjukkan pada huruf …
  • 91. 1. Pada sebuah pegas yang panjangnya 20 cm digantungkan beban bermassa 400 gram, panjang pegas menjadi 22,5 cm. Kemudian beban ditarik ke bawah sejauh 10 cm dan dilepaskan hingga terjadi gerak harmonik. Tentukan :  Konstanta pegas  frekuensi getaran pegas  periode getaran pegas 2. Sebuah bandul sederhana terdiri dari tali yang mempunyai panjang 40 cm dan pada ujung bawah tali digantungi beban bermassa 100 gram. Jika percepatan gravitasi 10 m/s2 maka periode dan frekuensi ayunan bandul sederhana adalah… 3. Lima pegas identik disusun seperti gambar. Setiap pegas memiliki konstanta 2.000 N.m–1. Jika pertambahan panjang secara keseluruhan 2 cm, maka besarnya energi potensial pegas adalah ....
  • 92. Tiga pegas identikmasing-masing dengan 30 = N.m-1 disusun seperti tampak pada gambar di samping. Saat beban m ditarik ke bawah sejauh 5 cm dari posisi keseimbangan untuk kemudian dilepaskan, pegas melakukan getaran harmonik sederhana. Berapakah perbandingan energi kinetik dan energi potensial pegas saat posisi beban m = 2 cm di bawah titik keseimbangan? A. 25 : 9 B. 25 : 16 C. 16 : 9 D. 5 : 4 E. 4 : 3
  • 93. Prediksi soal Tiga buah muatan disusun seperti pada gambar di bawah ini : Gaya Coulomb yang dialami muatan Q3 sebesar .... Kuprett solusion Tips: gambar dulu gaya Coulomb yang dialami Q3 karena Q1 dan Q3 Rumus yg diperlukan: .... 2 2 2 3 2 1 1 3 3 32 31 3      r Q Q k r Q Q k F F F F
  • 94.  Tidak terletak pada satu garis hubung Besar gaya coulomb yang dialami muatan A sesuai dengan aturan penjumlahan vektor adalah : +A +C -B  FA FAB FAC  Cos F F F F F AC AB AC AB 2 2 2     : sudut yang dibentuk oleh FAB dengan FAC
  • 95. Prediksi Soal 1) Tiga buah muatan listrik terletak di titik-titik sudut sebuah segitiga sama sisi ABC. Panjang sisi AB = AC = 5 cm, dan qA = - 10 µC, qB = qC= 10 µC. Besar gaya listrik yang bekerja pada muatan di titik A adalah…. 2) Perhatikan gambar di bawah ini Jika k = 9 x10 9 Nm – 2 , maka resultan gaya Coulomb yang bekerja pada muatan – 10 µC adalah . . . . A B C 6 µC – 8 µC 30 cm 30 cm – 10 µC
  • 96. Dua partikel masing-masing bermuatan qA = 1μC dan qB = 3 μC diletakkan terpisah sejauh 4 cm (k = 9 × 109 Nm2/C2). Besar kuat medan listrik di tengah-tengah qA dan qB adalah …. A. 6,75 × 107 N/C D. 3,60 × 107 N/C B. 4,50 × 107 N/C E. 2,25 × 107 N/C C. 4,20 × 107 N/C Pembahasan Perhatikan arah medan listrik yang terjadi di tengah-tengah kedua muatan tersebut. Karena EA dan EB berlawanan arah maka resultannya merupakan hasil pengurangan dari kedua medan listrik tersebut. Perhatikan gambar di bawah ini! Kuat medan listrik pada titik C sebesar ... (k = 9 × 109 N.m2.C−2). A. 5,0 × 109 NC−1 D. 3,1 × 1010 NC−1 B. 2,3 × 1010 NC−1 E. 4,0 × 1010 NC−1 C. 2,7 × 1010 NC−1 Pembahasan Titik C dipengaruhi medan listrik dari muatan A dan B. Karena muatan A dan B positif maka arah medan listrik di C dua-duanya ke kanan.
  • 97. Predikasi Soal 1) Perhatikan gambar rangkaian kapasitor berikut. Jika rangkaian tersebut dihubungkan dengan beda potensial 22 volt, maka besar muatan yang tersimpan dalam rangkaian kapasitor tesebut adalah.... Kapasitor Kapasitas Kapasitor keping sejajar: Muatan kapasitor: q = CV Kuat medan listrik: Energi tersimpan: Rangkaian Kapasitor:  Seri, berlaku:  Paralel, berlaku: A q d V E . 1 0 0       C q qV CV W 2 2 2 1 2 2    .... .... 1 .... 1 2 3 6          xV C q C C C total total s total s
  • 98. 2) Jika X dan Y pada rangkaian di samping dihubungkan ke batere 24 vot, maka besar muatan dan potensial pada kapasitor 12 F adalah …. 3) Perhatikan gambar rangkaian kapasitor di samping. Besar energi listrik dalam rangkaian adalah …. 4) Perhatikan grafik hubungan antara muatan q dan tegangan V sebuah kapasitor keping sejajar berikut ! Jika jarak antar keping 8,85 cm dano = 8,85 x 10-12 F.m-1, maka luas penampang keping sejajar adalah …
  • 99. Prediksi soal 1) Beberapa batere dan resistor dirangkai seperti gambar. Bila tegangan jepit pada hambatan 8 Ω adalah 4 volt, maka nilai R adalah…. R 4  8  12 v 3 v Ringkasan Materi Hukum ohm: V = I . R Hukum I Kirchoff “Jumlah kuat arus listrik yang masuk ke suatu titik sama dengan jumlah kuat arus listrik yang keluar” Imasuk = Ikeluar Hukum II Kirchoff “Di dalam sebuah rangkaian tertutup, jumlah aljabar gaya gerak listrik (E) dengan penurunan tegangan (IR) sama dengan nol” E= IR     .... 4 8 8 4 3 12 3 2 1 1 2 1             R R R R R R v E E iR E
  • 100. Perhatikan gambar Bila hambatan 12  diganti dengan hambatan lain yang besarnya 4 , perbandingan arus listrik yang mengalir pada rangkaian sebelum dan sesudah diganti adalah… A. 2 : 1 B. 2 : 3 C. 3 : 2 D. 11 : 12 E. 12 : 11 Beberapa baterai dan resistor dirangkai seperti gambar. Pada rangkaian diatas mengalir arus sebesar I1. Jika hambatan 4 diganti dengan hambatan 19  maka arus mengalir sebesar I2, maka nilai I2 adalah.... A. 0,1 I1 B. 0,2 I1 C. 0,3 I1 D. 0,4 I1 E. 0,5 I1
  • 101. 2) Pada gambar rangkaian di bawah, kuat arus listrik mengalir melalui hambatan 3  adalah …. Kuprett solusion Rs = 1 + 3 = … Rtot = 4 + Rp = …  Arus yang melalui hambatan yang dirangkai seri : dan 3) dari gambar di bawah ditanyakan i, VR, VAB ? Kuprett solusion ... 1   s P R ixR i ... 4 2   P ixR i A R3 E3 , r3 E2 , r2 B C D R2 R1 ..... ) ( .... ..... 2 1 1 1 3 3 2 2 1 1 3 2 1                     R r R i E V iR V r R r R r R E E E R E i AB R
  • 102. Prediksi soal Perhatikan gambar kawat berarus berikut ini. Jika arus yang mengalir 2 ampere dan titik P berjarak 0,5  cm dari kawat, maka induksi magnet pada titik P adalah....( µ0 = 4 π x 10-7 Wb A-1m-1 ) Kuprett solusion Dua buah kawat lurus panjang diletakkan sejajar pada jarak 2 cm satu sama lain dialiri arus seperti pada gambar. Jika arus yang mengalir pada kedua kawat sama besar 3A, maka induksi magnetik pada titik P adalah .... (o = 4.10-7 Wb.A-1.m-1) Kuprett solusion Tip: dengan aturan tangan kanan untuk kawat lurus diperoleh arah B yang ditimbulkan kawat pertama masuk bidang gambar dan arah B yang ditimbulkan kawat kedua juga masuk bidang gambar 7 0 4 .10 .... .... .... 2 2 .... iN x x B a x       .... .... 10 . 2 .... 10 . 4 .... 10 . 2 .... 10 . 4 7 7 7 7 2 1          x x x x B B BP     .... 10 . 2 2 2 1 1 7 2 1               a i a i B B BP
  • 103. .... 2 2 2 1 1 0    a ki a N i BP  Dua buah kawat berarus didekatkan seperti pada gambar. Besar induksi magnet di titik P adalah .... A. ( 4 + 2π ) .10-6 T B. ( 2 + 4π ) .10-6 T C. ( 1 + 2π ) .10-6 T D. ( 1 + 4π ) .10-6 T Kuprett solusion Tip: dengan aturan putaran baut diperoleh arah B yang ditimbulkan kawat melingkar masuk bidang gambar dan arah B yang ditimbulkan kawat lurus juga masuk bidang gambar. Bagaimana dengan induksi magnetik yang ditimbulkan kawat pada gambar berikut : i i P
  • 104. Sebuah kawat dialiri arus seperti gambar. Besar induksi magnet pada titik P adalah.... A. 0,4 x 10 -5 T B. 0.8 x 10 -5 T C. 1.0 x 10 -5 T D. 1,6 x 10 -5 T E. 2,4 x 10 -5 T Pembahasan: Perhatikan pernyataan yang berkaitan dengan induksi magnet 1) Arus yang mengalir dalam kawat 2) Permeabilitas ruang hampa 3) Luas penampang kawat 4) Massa jenis kawat Yang mempengaruhi besar induksi magnet disekitar kawat penghantar adalah... .
  • 105. Gaya magnet (gaya Lorentz)  Pada kawat berarus listrik: F = Bil sin   Pada muatan bergerak: F=BqV sin   Pada kawat sejajar: Arah kedua arus sama saling tarik-menarik Arah kedua arus berlawanan saling tarik-menarik  i B l A B U S i x x x x x x x x x x x x x x x x x x x x x x x x x x x x x x x x x x x x x x x x v a l i i F   2 2 1 0  2A 3A 5 cm arah arah
  • 106. 32. Gambar berikut menjelaskan arah gaya lorenzt pada muatan positif yang bergerak dalam medan magnet homogen. gambar yang benar ditunjukkan pada gambar nomor.... A. ( 1 ) B. ( 2 ) C. ( 3 ) D. ( 4 ) E. ( 5 ) Pembahasan:
  • 107. Fluks magnetik melalui sebuah loop dinyatakan dengan persamaan ɸ =( t2 - 8t) x 10-2 wb, dengan t dalam sekon. Tentukan besar ggl induksi yang dihasilkan jika jumlah lilitan komparan 400 lilitan dan t = 7 sekon! A. 4 volt B. 12 volt C. 16 volt D. 24 volt E. 30 volt Fluks magnetik yang dirangkul oleh kumparan 100 lilitan berubah memenuhi persamaan fluks:  = (3t2 + 2t –1) weber. Pada saat t = 2 s, besar GGL induksi magnetik di ujung-ujung kumparan adalah …
  • 108. Tranformator (menjelaskan teori)  Bentuk dan Simbol Transformator (Trafo) terdiri dari kumparan kawat yang terisolasi yaitu kumparan primer dan kumparan sekunder (biasanya lebih dari satu lilitan) yang dililitkan pada sebuah besi yang dinamakan dengan Inti Besi (Core). Sedangkan Inti besi pada terbuat dari kumpulan lempengan-lempengan besi tipis yang terisolasi dan ditempel berlapis-lapis untuk mempermudah jalannya Fluks Magnet yang ditimbulkan oleh arus listrik kumparan serta untuk mengurangi suhu panas yang ditimbulkan.
  • 109.  Prinsip Kerja Transformator (Trafo) 1) Ketika kumparan primer dialiri arus AC (bolak-balik) maka akan menimbulkan fluktuasi medan magnet atau fluks magnetik disekitarnya. Kekuatan Medan magnet (densitas Fluks Magnet) tersebut dipengaruhi oleh besarnya arus listrik yang dialirinya. Semakin besar arus listriknya semakin besar pula medan magnetnya. 2) Fluktuasi medan magnet yang terjadi di sekitar kumparan pertama (primer) akan menginduksi GGL (Gaya Gerak Listrik) dalam kumparan kedua (sekunder) dan akan terjadi pelimpahan daya dari kumparan primer ke kumparan sekunder. Dengan demikian, terjadilah pengubahan taraf tegangan listrik baik dari tegangan rendah menjadi tegangan yang lebih tinggi maupun dari tegangan tinggi menjadi tegangan yang rendah.
  • 110. 34. Dalam suatu percobaan tarfo diperoleh data sebagai berikut. A. 1500 volt B. 2500 volt C. 3000 volt D. 4500 volt E. 6000 volt Pembahasan: IP IS VP VS NP NS 4 3 P 300 1200 R
  • 111.
  • 112.  Rangkaian R murni  Rangkaian L murni  Rangkaian C murni
  • 113. Prediksi soal 1. dari gambar di bawah ini ditanyakan Z atau i 2. Rangkaian RLC seri seperti pada gambar Bila saklar S ditutup, beda potensial antara titik A dan B adalah.... 400  1 H 10 µF 12 V / 200 rad.s-1
  • 114. 1) Rangkaian RLC disusun seperti pada gambar Impedansi pada rangkaian tersebut adalah.... A. 1200 ohm B. 1000 ohm C. 500 ohm D. 200 ohm E. 125 ohm 2) Di bawah ini adalah diagram fasor dari rangkaian RLC seri yang dihubungkan dengan tegangan bolak-balik. Rangkaian yang bersifat kapasitif ditunjukkan pada gambar…
  • 115. 2) Grafik sinusoida berikut ini menunjukan hubungan V- t dan I – t dari rangkaian seri RLC. Rangkaian yang bersifat kapasitif adalah . . A. D. B. E. E. V, i t 0 V i V, i t 0 V i V, i 0 V i V, i 0 V i V, i t 0 V i
  • 116. 1) Teori atom a) Teori atom Dalton Teori atom Dalton dikemukakan berdasarkan hukum kekekalan massa dan hukum perbandingan tetap. Teori atom Dalton dikembangkan mulai dari 1803 – 1808, dan didasarkan atas tiga asumsi, yaitu :  Setiap unsur kimia tersusun atas partikel – pertikel kecil yang tidak dapat dihancurkan dan dipisahkan yang disebut atom (Selama mengalami perubahan kimia, atom tidak dapat diciptakan dan dimusnakan).  Semua atom dari suatu unsur mempunyai massa dan sifat yang sama, tetapi atom – atom dari unsur yang berbeda dengan atom – atom dari unsur yang lain, baik massa maupun sifat – sifatnya berlainan.  Dalam senyawa kimiawi, atom – atom dari unsur yang berlainan melakukan ikatan dengan perbandingan angka sederhana Kelebihan : kelebihan dari teori atom Daton adalah sebagai berikut :  Dia adalah orang pertama yang melibatkan kejadian kimiawi seperti halnya kejadian fisis dalam merumuskan gagasannya tentang atom.  Dia medasarkan asumsinya pada data kuantitatif, tidak menggunakan pengamatan kualitatif atau untung – untungan. Kekurangan : kekurangan dari teori Dalton adalah tidak dapat menerangkan hubungan antara larutan senyawa dan daya hantar listrik.
  • 117.  Thompson memperbaiki kelemahan dari teori atom Dalton dan mengemukakan teori atomnya yang dikenal dnegan nama teori atom Thompson.  Teori ini menyatakan bahwa “atom merupakan bola pejal yang bemuatan positif dan didalamnya tersebar elektron yang bermuatan negatif. Model atom Thompson ini dapat anda bayangkan sebagai tori kismis dimana didalamnya tersebar elektron. Dengan ungkapan Thompson tersebut, maka dapat disimpulkan bahwa teori atom Thompson adalah netral. Kelebihan : kelebihan model atom Thompson, yaitu berhasil menemukan particle yang lebih kecil dari atom yaitu elektron. Elektron ini ditemukan melalui percobaan sinar katoda. Kekeurangan : kekurangan dari model atom Thompson yaitu tidak dapat menerangkan susunan muatan positif dan negatif dalam atom tersebut
  • 118.  Teori atom Rutherford muncul berdasarkan eksperimen hamburan sinar alfa dari uranium. Kesimpulannya adalah : 1) Atom tersusun atas inti atom yang bermuatan positif dan elektron yang bermuatan negatif yang mengelilinginya 2) Semua proton terkumpul dalam inti atom, dan menyebabkan inti atom bermuatan positif. 3) Sebagian volume atom adalah ruang kosong. hampir semua massa atom terpusat pada inti atom yang sangat kecil. Jari – jari atom sekitar 10-10m sedangkan jari – jari inti atom sekitar 10-15 m. 4) Jumlah proton dalam inti atom sama dengan jumlah elektron yang mengelilingi inti atom, sedangkan atom bersifat netral. Kelebihan : kelebihan teori atom ini mampu menjelaskan susunan positif dan negatif pada ato serta dapat menjelaskan bahwa atom tidak berbentuk bola pejal dan sebagian besar volumenya merupakan ruang kosong. Kekurangan : kekurangan teori ataom ini yaitu tidak dapat menjelaskan mengapa elektron tidak dapat jatuh kedalam inti atom
  • 119. Niels Bohr, melakukan percobaan spektrum hidrogen untuk memperbaiki teori atom Rutherford. Hasil percobaan Bohr : 1) elektron – elektron mengelilingi inti atom yang terdiri dari Proton dan Neutron pada lintasan – lintasan tertentu yang disebut kulit eletron atau tingkat energy. (Tingkat energy paling rendah adalah kulit elektron yang terletak paling dalam, semakin keluar maka semakin besar pua nomor kulitnya dan semakin tinggi tingkat energinya). 2) Menurut teori atom bohr, kulit pertama dalam atom di nama dengan kulit K (n=1), L(n=2), M,(n=3) dan seterusnya. Dalam model atom bohr ini dikenal istilah konfigurasi elektron, yaitu susunan elektron pada masing – masing kulit. Konfigurasi pada teori atom bohr dapat dicari dengan persamaan 2n2 dimana n adalah nomor kulit atom. Misal, pada kulit L, karena di kulit L ini n = 2, maka jika dimasukkan ke persamaan adalah 2.22 = 2.4 = 8 Jadi elektron maksimal yang dapat menempati kulit atom kedua sebanyak 8 elektron.(lebih jelasnya nanti akan dipelajari pada bab konfigurasi elektron) Kelebihan model atom Bohr Atom terdiri dari beberapa kulit/subkulit untuk tempat berpindahnya elektron dan atom membentuk suatu orbit dimana inti atom merupakan positif dan disekelilingnya terdapat elektron. Kelemahan model atom Bohr a. Tidak dapat menjelaskan efek Zeeman dan efek Strack. b. Tidak dapat menerangkan kejadian-kejadian dalam ikatan kimia dengan baik, pengaruh medan magnet terhadap atom-atom, dan spektrum atom yang berelektron lebih banyak.
  • 120. Model atom mekanika kuantum didasarkan atas :  Electron bersifat gelombang dan partikel, oleh Louis de Broglie pada tahun 1923.  Persamaan gelombang dalam atom, oleh Erwin Schrodinger pada tahun 1926.  Asas ketidakpastian oleh Werner Heisenberg pada tahun 1927. Menurut teori mekanika kuantum, elektron tidak bergerak pada lintasan tertentu. Berdasarkan model atom tersebut, maka model atom mekanika kuantum adalah sebagai berikut :  Atom terdiri atas inti atom yang mengandung proton dan neutron. Dan electron – electron yang mengelilingi inti atom berada pada orbital – orbital tertentu yang membentuk kulit atom. Hal ini disebut konsep orbital.
  • 121. 36. Yang merupakan karakteristik dari model atom Bohr adalah.... A. elektron berputar mengelilingi inti seperti planet mengelilingi matahari dalam tatasurya B. Atom bersifat netral sehingga jumlah muatan negatif sama dengan muatan positif C. elektron akan memancarkan / menyerap energi jika elektron mengalami transisi kulit D. spektrum yang dihasilkan elektron dalam mengitari inti bersifat kontinyu E. lintasan elektron dalam mengitari inti berupa setengah lingkaran Pembahasan:
  • 122.
  • 123. Efek foto listrik  Peristiwa efek fotolistrik yaitu terlepasnya elektron dari permukaan logam karena logam tersebut disinari cahaya.  Menurut Einstein energi yang dibawa foton dalam bentuk paket, seluruhnya diberikan pada elektron, sehingga foton tersebut lenyap. Oleh karena elektron terikat pada energi ikat tertentu, maka untuk melepaskan elektron dari energi ikatnya diperlukan energi minimal sebesar energi ikat elektron yang disebut fungsi kerja (Wo) atau energi ambang yang besarnya Wo tergantung pada jenis logam yang digunakan.
  • 124.
  • 125.
  • 126.
  • 127.
  • 128.
  • 129.  Pernyataan yang berkaitan dengan peristiwa efek foto listrik adalah.... A. Ikatan atom pada logam yang disinari mudah putus sehingga elektron dapat terlepas B. Lepasnya elektron dari ikatan logam disebabkan oleh cahaya yang memiliki frekeunsi ambang C. Intensitas cahaya yang digunakan mampu memutuskan ikatan logam D. Frekuensi cahaya yang digunakan untuk menyinari logam harus kecil E. Energi yang dihasilkan sama dengan energi ambang
  • 130. Prediksi soal 1. Sebuah roket yang sedang diam dibumi panjangnya 10 m. Jika roket bergerak dengan kecepatan 0,6 c, maka menurut pengamat di bumi panjang roket tersebut selama bergerak adalah …. Kuprett solusion 2. A dan B adalah sepasang anak kembar. A berkelana di antariksa dengan pesawat berkecepatan 0,6 c. ketika A kembali di bumi, menurut B perjalanan A sudah berlangsung 20 tahun. Maka menurut A perjalanan yang telah ditempuh memakan waktu … 3. Cepat rambat cahaya di udara c, agar panjang benda yang diamati susut menjadi 80% dari ukuran benda yang teramati oleh pengamat yang diam, maka cepat rambat pengamat yang bergerak terhadap benda tersebut adalah ... . A. 0,95 c B. 0,70 c C. 0,80 c D. 0,60 c E. 0,25 c ..... 4 5 10 0     L L
  • 131. Prediksi soal Diketahui massa proton = 1,0078 sma, massa neutron = 1,0086 sma, massa = 4,002 sma massa = 7,0160 sma, massa = 7,0169 sma, dan 1 sma = 931 MeV. Hitung Energi ikat inti He, Li dan Be ! Kuprett solusion E = Δm x 931 =(Zmp + (A-Z)mn - minti)x 931 MeV Langkah penyelesaian:
  • 132. Inti atom yang terbentuk memenuhi reaksi fusi berikut ini : Diketahui : Massa 1H2= 2,0141 sma Massa 1H3 = 3,0160 sma Massa = 4,0026 sma Massa = 1,0086 sma 1 sma = 931,5Mev Nilai E (energi yang dihasilkan) pada reaksi fusi tersebut adalah .... Kuprett solusion Langkah penyelesaian:   MeV m m E reaksi hasil pereaksi 931     Suatu proses fisi pada Reaktor Nuklir mengikuti persamaan : Jika massa neutron 1,009 sma, massa inti = 235,040 sma, massa inti = 140,910 sma, massa inti = 91,930 sma dan 1 sma = 931 MeV. Maka besar energy Q yang dibebaskan pada reaksi fisi tersebut adalah …..
  • 133. 39. Perhatikan reaksi inti berikut 0n1 + 12A25  4X9 + 7Y17 + Q Data massa adalah sebagai berikut. 0n1 = 1,0008 sma 12A25 = 26,0012 sma 4X9 = 9,0500 sma 7Y17 = 17,1500 sma Jika 1 sma setara dengan 931 MeV, maka dalam reaksi .... A. Dihasilkan energi 746,7 MeV B. Diperlukan energi 746,7 MeV C. Dihaslkan energi 186,2 MeV D. Diperlukan energi 186,2 MeV E. Dihasilkan energi 93,1 MeV
  • 134. Perhatikan reaksi inti berikut ini. Nilai X, Y dan Z adalah ... .
  • 135. 40. Radio isotop adalah isotop yang bersifat radioaktif sehingga jejaknya dapat dikenal. Berikut ini adalah pemanfaatan radio isotop dalam bidang kedokteran: 1) Co-60 : terapi radiasi gamma untuk pengobatan kanker 2) C-14 : Untuk menentukan umur fosil 3) Galium-67 : untuk pengobatan kelenjar getah bening 4) Sb-124 : untuk mendeteksi adanya kebocoran pipa- pipa industri. Pernyataan yang benar adalah …. A. 1 dan 2 B. 2 dan 3 C. 1 dan 3 D. 2 dan 4 E. 1 dan 4
  • 136. Perhatikan diagram di bawah ini N = massa zat radioaktif mula-mula (dalam gram) t = waktu selama peluruhan (dalam tahun) jika zat radioaktif disimpan selama 6 tahun, maka zat yang tersisa adalah .... T t t T t t m m N N T                2 1 2 1 693 , 0 0 0 
  • 137. t (sekon) m (gram) 100 50 12,5 0 6 9 Perhatikan grafik peluruhan massa suatu zat radioaktif di bawah Berdasarkan grafik konstanta peluruhan (λ) zat radioaktif adalah …
  • 138. Perhatikan gambar grafik peluruhan berikut ini. Urutan aktivitas radiasi dari yang kecil ke besar adalah …
  • 140. Lampiran: Berikut ini adalah efek samping yang diteliti pada manusia yang terpapar langsung gelombang mikro, tanpa harus mengkonsumsi zat makanan yang diradiasi.  Kerusakan medan energi kehidupan (Orang yang dekat dengan oven microwave saat dioperasikan dalam jangka lama akan mengalami gangguan medan energi dalam kehidupan mereka).  Mengurangi energi dari sel (Paralel tegangan seluler individu menggunakan alat akan mengalami penurunan terutama serum limfatik dalam darah mereka).  Metabolisme tidak stabil (Energi eksternal yang diaktifkan berpotensi dalam pemanfaatan makanan menjadi tidak stabil dan mengalami degenerasi).  Kerusakan sel (Selama proses katabolik pada internal Membran sel ke serum darah, pada proses pencernaan berpotensi merosot dan tak stabil).  Sirkuit Otak (Impuls listrik pada persimpangan dari otak berpotensi besar akan merosot dan rusak).  Sistem Saraf (Saraf/sirkuit listrik akan merosot dan memecah, sementara simetri medan energi yang hilang dalam neuropleksus (pusat saraf) pada otonom sistem saraf pusat dan belakang).  Kehilangan Kekuatan Bioelectric (Kekuatan bio listrik pada reticular ascending (netlike) yang mengaktifkan sistem (sistem yang mengontrol fungsi kesadaran) akan menjadi tak seimbang dan kehilangan arus yang tepat).  Kehilangan Energi Vital (Manusia, hewan dan tumbuhan yang berada dalam radius 500m dari peralatan gelombang micro yang dioperasikan akan mengalami kehilangan energi penting dari kumulatif jangka panjang . Apalagi hanya dalam beberapa meter).  Gugup dan mengalami kerusakan sistem limfatik (Dalam jangka panjang “Deposito” residu magnetik akan terjadi di seluruh sistem saraf dan limfatik. Ketidakseimbangan Hormon.Produksi hormon dan pemeliharaan keseimbangan hormonal pada laki-laki dan perempuan menjadi tidak stabil dan terganggu).  Gangguan gelombang otak (Tingkat gangguan pada gelombang pola sinyal alpha, delta dan theta ternyata lebih tinggi dari normal).  Gangguan Psikologis (Karena gelombang otak menjadi kusut, efek psikologis yang negatif juga akan dihasilkan. Hal ini termasuk hilangnya kemampuan memori dan konsentrasi, ambang emosional yang tertekan, perlambatan proses intellective dan tidur yang terganggu, dengan persentase yang lebih tinggi dari individu yang mengalami efek kisaran emisi berkelanjutan microwave, baik dari memasak ataupun dari transmisi stasiun).